Download as pdf
Download as pdf
You are on page 1of 728
PROBABILITY AND STATISTICS & COMPLEX VARIABLES [For JNTU (H) B.Tech. Second Year I Semester Students of Mechanical, Mechatronics , Mining, Petroleum, Metallurgics! and Materials Engineering as per New Syllabus (R18)| [Including Chapter-Wise Previous GATE Questions] Dr. T.K.V. IYENGAR Dr. B. KRISHNA GANDHI MA., PhD, MSe., PhD. Former Professor of Mathematics Former Professor of Mathematics National Institute of Technology JINTU, Hyderabad Warangal Former Vice Chancellor Jawaharlal Nehru Technological University Anantapur. S. RANGANATHAM PRASAD MSc, MPhil MSc, PEDCA, PhD Former Head of the Dept. of Mathematics Principal invaharbharan College Deparment of ecnia cation Koval Govt of AP = == s.cHaND S Chand And Company Limited (An ISO 9001 : 2008 Company) RAM NAGAR, NEW DELHI - 110 055 S Chand And Company Limited (ISO 9001 Certified Company) S,SHAND ead ofice: 7361, RAM NAGAR, QUTAB ROAD, NEW DELHI - 110055 empowering minds Phone: 23672080-81-82, 66672000 Fax: 91-11-23677446 x a ranch www.schandpublishing.com; e-mail: info@schandpublishing.com ‘Ahmedabad Ph: 27541965, 27542369, ahmedabad@schandpublishing.com Bengaluru Ph: 22268048, 22354008, bangalore@schandpublishing.com Bhopal Ph: 4209587, bhopal@schandpublishing.com Chandigarh: Ph: 2625366, 2625546, 4025418, chandigarh@schandpublishing.com Chennai Ph: 28410027, 28410058, chennai@schandpublishing.com Coimbatore: Ph: 2329620, 4217136, colmbatore@schandpublishing, com (Marketing Office) Cuttack Ph: 2382580, 2392581, cuttack@schandpublishing.com Dehradun Ph: 2711101, 2710861, dehradun@schandpublishing.com Guwahati Ph: 2738811, 2735640, guwahati@schandpublishing.com Hyderabad Ph: 27550194, 27550195, hyderabad@schandpublishing.com Jaipur Ph: 2219176, 2219176, jalpur@schandpublishing,com Jalandhar Ph: 2401630, jalandhar@schandpublishing.com Kochi Ph: 2809208, 2808207, cochin@schandpublishing.com Kolkata : Ph: 23353014, 23357458, kolkata@schandpublishing.com Lucknow Ph: 4065646, lucknow@schandpublishing.com ‘Mumbai Ph: 22680881, 22610885, 22610886, mumbai@schandpublishing.com Nagpur Ph: 2720523, 2777666, nagpur@schandpublishing.com Patna Ph: 2300489, 2260011, patna@schandpublishing.com Pune Ph: 64017298, pune@schandpublishing.com Raipur Ph: 2443142, raipur@schandpublishing.com (Marketing Office) Ranchi Ph: 2364178, ranchi@schandpublishing.com ‘Sabibabad Ph: 2771235, 2771238, delhibr-sahibabad@schandpublishing.com © 2008, $ Chand And Company Limited All rights reserved. No part ofthis publication may be reproduced or copied in any material form (including photocopying or storing it in any medium in form of graphics, electronic or mechanical means and whether ‘r not transient or incidental to some other use of this publication) without written permission of the copyright ‘owner. Any breach of this will entail legal action and prosecution without further notice. Jurisdiction : All disputes with respect to this publication shall be subject to the jurisdiction of the Courts, Tribunals and Forums of New Delhi, india only First Edition 2008 ‘Subsequent Editions and Reprints 2010, 2011, 2012, 2013, 2014, 2015, ‘Seventh Revised Eaition 2015 Eighth Revised Eaition 2020 1978-93-5283-748.9 Product Code: H6PSC61MATH10ENAH20S PRINTEDININOIA By Vikas Publishing House Pvt. Ltd., Plot 20/4, Site-V, Industrial Area Sahibabad, Ghaziabad-201010 and Published by S Chand And Company Limited, 7361, Ram Nagar, New Delhi-110 056. PREFACE ‘We are happy to bring out the First Edition of the book “Probability and Statistics & Complex Variables”. This text book has been written strictly according to the latest syllabus (R-18) prescribed by Jawaharlal Nehru Technological University, Hyderabad for B. Tech, II year First Semester students of Mechanical, Mechatronics , Mining, Petroleum, Metallurgics! and Materials Engineering. An important feature is the inclusion of Previous GATE Questions at the end of each of chapter for the benefit of students. Questions from latest question papers of J.N.T.U., Hyderabad have been inserted at proper places. The Objective type questions have been given at the end of each unit, We hope this edition will meet the requirements of students and teachers. We gratefully welcome suggestions from readers for improvement of the book. We are thankful to the Management Team and the Editorial Department of S Chand And Company Limited, New Delhi forall the help and support in the publication of this book. — AUTHORS SYLLABUS (R-18) (ove: Acadamie Year 2018-19) JNTU - HYDERABAD (MA301BS) PROBABILITY AND STATISTICS & COMPLEX VARIABLES (For B. Tech. II year I Semester Mechanical, Mechatronics , Mining, Petroleum, Metallurgics! and Materials Engineering Students) UNIT-1 : Basic Probability + Probability spaces, conditional probability, independent events, and Bayes’ theorem. Random variables : Discrete and continuous random variables, Expectation of, Random Variables, Moments, Variance of random variables. UNIT--II : Probability Distributions ¢ Binomial, Poisson, evaluation of statistical parameters for these distributions, Poisson approximation to the binomial distribution. Continuous random variables and their properties, distribution functions and density functions, Normal and exponential, evaluation of statistical parameters for these distributions UNIT- Ill : Testing of Hypothesis + Test of significance : Basic of testing of Hypothesis. Null and altemate Hypothesis, types of errors, level of significance, critical region. Large sample test for single propotion, difference of proportions, single mean, difference of means; small sample tests: Test for single mean, difference of means and test for ratio of variances. UNIT-IV : Complex Variables (Differentiation) ¢ Limit, Continuity and Differentiation of Complex functions, Analyticity, Cauchy- Riemann equations (without proof), finding harmonic conjugate; elementary analytic functions (exponential, trigonometric, logarithm) and their properties. UNIT-V_ : Complex Variables (Integration) + Line integral, Cauchy’s theorem, Cauchy’s integral formula, zeros of analytic functions, Singularities, Taylor's series, Laurent’s series; Residues, Cauchy Residue theorem, Conformal mappings, Mobius transformations and their properties. CONTENTS UNIT -1 Basic Probability 1. Review of Probability 3-42 2. Random Variables 43 - 125 UNIT -il_ Probability Distributions 3. Probability Distributions 129 - 242 UNIT - Ill Testing of Hypothesis 4. Tests of Hypothesis (For Large Samples) 245 - 311 5. Small Samples 312 - 379 UNIT - IV_ Complex Variables (Differentiation) 6. Functions of A Complex Variable 383 - 469 UNIT - V_ Complex Variables (Integration) 7. Complex Integration 473 - 562 Complex Power Series 563 - 620 9. Contour Integration 621- 672 10. Conformal Mapping 673 - 700 © Statistical Tables 703 - 119 1. Review of Probability + Sample Space and Events * Axioms of Probability * Conditional Probability * Baye’ Theorem 2. Random Variables * Random Variables * Distribution Function * Discrete and Continuous Probability Distributions ¢ Mathematical Expectation and Properties * Central Moments ¢ Raw Moments bese see eee eee eee eed CHAPTER - 1 REVIEW OF PROBABILITY 1.1. INTRODUCTION The theory of probability is one of the most useful and interesting branches of modern mathematics. It is becoming prominent by its application in many fields of learning, such as Insurance, Statistics, Biological Sciences, Physical Sciences, Engineering, etc. An Italian mathematician, Galileo (1564 - 1642), attempted a quantitative measure of probability while dealing with some problems related to gambling. In the middle of 17th Century, two French mathematicians, Pascal and Fermat, laid down the first foundation of the mathematical theory of probability while solving the famous ‘Problem of Points’ posed by Chevalier-De- Mere. Other mathematicians from several countries also contributed in no small measure to the theory of probability. Outstanding of them were two Russian mathematicians, A. Kintchine and A. Kolmogoroff, who axiomised the calculus of probability. If an experiment is repeated under similar and homogencous conditions, we generally ‘come across two types of situations. (H The net result, what is generally known as ‘outcome’ is unique or certain, (ii) The net result is not unique but may be one of the several possible outcomes. The situations covered by (i) are known as ‘deterministic’ or ‘predectable’ and situations covered by (ii) are known as ‘probabilistic’ or ‘unpredictable’. ‘Deterministic’ means the result can be predicted with certainty. For example, if r is the 4 nr? which gives uniquely the volume of radius of the sphere then its volume is given by V the sphere There are some situations which do not lend themselves to the deterministic approach and they are known as ‘Probabilistic’ For example, by looking at the sky, one is not sure whether the rain comes or not. In such cases we talk of chances or probability which can be taken as a quantitative measure of certainity. We will now introduce the concept of probability with definitions. 4.2. RANDOMEXPERIMENT [NTU 2007 (SUP)] Definition. If an ‘experiment’ is conducted, any number of times, under essentially identical conditions, there is a set of all possible outcomes associated with it. Ifthe result is not certain and is anyone of the several possible outcomes, the experiment is called a random trial or a random experiment. The outcomes are known as elementary events and a set of outcomes is an event. Thus an elementary event is also an event. 4 Probability and Statistics & Complex Variables 4.3 EQUALLY LIKELY EVENTS Events are said to be equally likely when there is no reason to expect anyone of them rather than anyone of the others. e.g. When a card is drawn from a pack, any card may be obtained. In this trial, all the 52 elementary events are equally likely. 1.4 EXHAUSTIVE EVENTS All possible events in any trial are known as exhaustive events. e.g. 1. In tossing a coin, there are two exhaustive elementary events, viz. head and tail. 2. In throwing a die, there are six exhaustive elementary events (i.e.), getting 1 or 2 or 3 or 4 or Sor 6, 3. In drawing 3 balls out of 9 balls in a box, there are 9, exhaustive elementary events. 4.5 MUTUALLY EXCLUSIVE EVENTS [NTU Aug 2007 Sup] Events are said to be mutually exclusive, if the happening of any one of the events in a trial excludes the happening of any one of the others i.e., if no two or more of the events can happen simultaneously in the same trial 1.6 CLASSICAL DEFINITION OF PROBABILITY Ina random experiment, let there be n mutually exclusive and equally likely elementary events. Let E be an event of the experiment. If m elementary events form event E (are favourable to £), then the probability of E (Probability of happening of E or chance of ), is defined as ‘Number of elementary events in E ~ Total number of elementary events in the random experiment P(E) alk If E denotes the event of non-occurrence of E, then the number of elementary events in E is n—m and hence the probability of E (non-occurrence of B) is m - ~ 7 =1- P= P+ PE)=1 n Since m is a non-negative integer, m isa positive integer and m 1<2 P(A) 1 1 > 7 5 OLVED Aig [BRDIIEEA What isthe probability for a leap year to have 52 Mondays and 53 Sundays? Solution: A leap year has 366 days i.e., 52 weeks and 2 days. These two days can be any one of the following 7 ways : (@) Mon & Tue (if) Tues & Wed (iii) Wed & Thurs (iv) Thurs & Fri (v) Fri & Sat (vi) Sat & Sun (vii) Sun & Mon Let E be the event of having $2 Mondays and 53 Sundays in the year. Total number of possible cases is n= 7 Number of favourable cases to E is m (Sat & Sun is the only favourable case) 1 . PE) = as Five digit numbers are formed with 0, 1, 2,3, 4 (not allowing a digit being repeated in any number). Find the probability of getting 2 in the ten’s place and 0 in the units place always. Solution: Total number of 5 digit numbers using the digits 0, 1, 2, 3, 4 is =4x4x3x2%1=96 (or) 5!-41=96 . Let E be the event of getting a number having 2 in 10’s place and 0 in the units place. So the number of numbers favourable to E= m = 3.2.1.1.1=6 m_ 61 PE)= 56 ~ 16 In aclass there are 10 boys and 5 girls. A committee of 4 students is to be selected from the class. Find the probability for the committee to contain at least 3 girls. Solution: A committee of 4 students out of 15 can be formed in ‘Sc, ways ie, n=BCq Let £ be the event of forming a committee with at least 3 girls. Now the committee can have | boy, 3 girls or no boy, 4 girls. So the number of ways of forming the committee = The number of favourable ways to E = 9, x53 + 9g x Cy =100+5=105 m _ 105 PO= 5 “te = 0.0769 6 Probability and Statistics & Complex Variables A class consists of 6 girls and 10 boys. If a committee of 3 is chosen at random from the class, find the probability that (i) 3 boys are selected (ii) exactly 2 girls are selected. [JNTU 20088 (Set No.2)] Solution: Total number of students = 16 n(S) = no. of ways of choosing 3 from 16 = "6c, (i) Suppose 3 boys are selected. This can be done in 'c, ways. Here n(E)= "c, mE P(E) =The probability that 3 boys are selected -2 10, = Cr, MOK9x8 _ 3 yg, We, 16xISx14 14 (ii) Suppose exactly 2 girls are selected. Then n(E) = §Cy xc, 65x10 Cx Mo, 15 “Ox _ BS oy iG, 5g = 02678 |A and B throw alternately with a pair of ordinary dice. A wins if he throws 6 before B throws 7 and B wins if he throws 7 before A throws 6. IfA begins, show that his chance of winning is 30/61 [NTU April 2004 (Set No. 3)] Solution; When two dice are thrown , we have n(s) = 36 g- 2 = The probability of A throwing ‘6’ = 36 be P(A) = 36 is and is given by The probability of 4 not throwing 5331 PA) =1-PU)=1~ 36 = 35 6 61 The probability of B throwing ‘7’ = =~ i.e., P(B)= > = 36 36-6 The probability of B not throwing 7 is P(B) =1-p@)=1-t=3 Chances of winning of ‘A’ is = P(A)+P(A )P(B)P(A) +P(A )P(B)P(A)P(B P(A) + - (2) +(Beded) (Bd BES) ~ (36) * (36 *6 36) * (36% 6 *36*6~ 36) * a “(3 2).(2 a 36 | "(36% 6) "(36 61 Review of Probability 7 A, Band C in order toss a coin. The first one to toss head wins the game. ‘What are the probabilities of winning, assuming that the game may continue indefinitely. [NTU August 2007S, (H) Apr. 2012 (Set No. 1), Dec. 2012] Solution: A may win in the first round with probability 1/2 He may win in the second round, after all of them failed in the first round, with the 3 6 probability (3) + in the third round with probability = (3) 4 and so on. By Addition theorem, the chance of A’s success is i J Q)-4- 2 272 2)°2 ~ u(y «(eye ae ap 2 2 a Tam | 1¢ series is in G P.] 1 4 aa & 5 g 2 é " (Series is in GP.) ‘Two cards are selected at random from 10 cards numbered 1 to 10. Find the probability that the sum is even if (i) The two cards are drawn together. (ii) The two cards drawn one after other with replacement. [JNTU Nov. 2006 (Set No. 3)] Solution: (i) Suppose two cards are drawn at a time. ., Number of ways of drawing two cards at a time from 10 cards = !°c, = 45. For the sum on both the cards to be even both the cards should be even numbered or both the cards should be odd numbered. ‘Two even numbered cards can be choosen from five even numbered cards in 5C, = 10 ways :. Total no. of favourable outcomes = 10 + 10 = 20. 8 Probability and Statistics & Complex Variables ., Required probability 59 (ii) Suppose the two cards are choosen one after another with replacement. This can be done in 10 = 10 = 100 ways. For the sum to be even both the cards must be even or both the cards must be odd. Number of ways of selecting two even cards = 5 x 5 = 25. Similarly, number of ways of selecting two odd cards = 5 x 5 = 25. 25+25 1 Required Probability = 55~ = > ‘A box contains n tickets marked 1 through n. Two tickets are drawn in succession without replacement. Determine the probability that the number on the tickets are consecutive integers. [INTU 2004S (Set No. 4), Feb 2008 S (Set No. 3)] Solution _: The box contains n tickets marked | through n. The number of ways in which two tickets can be drawn without replacement =n(n-1) The total number of favourable cases in which the numbers on the tickets are consecutive integers =n -1. Required probability = 5 (, ‘Two cards are selected at random from 10 each numbered 1 to 10. Find the probability that the sum is odd if (Two cards are drawn. (ii) Two cards are drawn one after another with replacement, (iii) Two cards are drawn one after another without replacement [JNTU 2003 S] Solution ; (i) Two cards can be drawn at a time from 10 cards in °c, = 45 ways Let E, denote the event of the two cards are such that the sum is odd ‘We must have one card even and another odd. Number of ways of doing it = 5c, x °C, =25 R probability = 25 = 3 equired probability = 32 = 5 (ii) Let £, = The sum is even when two cards are drawn one after another with replacement. The no. of favourable cases = 50. The no. of ways in which two cards can be drawn one after another with replacement =!¢, x!c, =100 30 Required probability = 755 = Review of Probability 9 (iii) The no. of favourable cases = 50 The number of cases that the two cards can be drawn one after another without replacement = !¢, x°C, =90 :. Required probability = 2 ar Determine the probability for each of the following events : Anon- defective bolt will be found if out of 600 bolts already examined 12 were defective. [INTU Dee. 2005, April 2006 (Set No. 1)] 12 1 600 50° :. The probability of finding a non-defective bolt = P(D) = 1 ~ P(D) Solution ; The probability of defective bolt, P(D) ees ~ 50 ~ 50 1.7 AXIOMATIC DEFINITION OF PROBABILITY The classical definition of probability breaks down when we do not have a complete priori analysis i.e., when the outcomes of the trial are not equally likely or when the total number of trials is infinite or when the enumeration of all equally likely events is not possible. So the necessity of the following definition viz., statistical or empirical definition of probability. 1.8 VON MISES’S STATISTICAL DEFINITION OF PROBABILITY Let a trial be repeated any number of times under essentially identical conditions and let Evbe an event of it. The ratio of the number of times (m) the event E happens to the number of trials (n) ie., m/n is called the relative frequency of the event E and is denoted by R(E). Then the probability P(E) of the event E is the limit approached by R(E) as the number of trials (n) increases indefinitely, it being assumed a unique limit value exists. Symbolically, we write = Lt, R(E) = PEE) ne The statistical definition of probability, although is of great use from practical point of view, is not conductive for mathematical approach since an actual limiting number may not really exist. Hence another definition is thought of based on axiomatic approach. This definition leads to the development of calculus of probability. 14.9 SIMPLEEVENT [JNTU 2007 Sup] An event in a trial that cannot be further split is called a simple event or an elementary event, 4.10 SAMPLE SPACE [JNTU 2007 Sup] The set of all possible simple events in a trial is called a sample space for the trial. Each element of a sample space is called a sample point. Any subset of a sample space is an event. Itis generally denoted by E. Thus a simple event is a sample point. Sample space is denoted by S. eg : Two coins are tossed. 10 Probability and Statistics & Complex Variables Then the possible simple events of the trial are HH, HT, TH, TT. .. The sample space of the trial = S= (HH, HT, TH, TT} If an event (E) is that either two heads or two tails appear then E = {HH, TT}. Clearly the elements of £, are sample points and E < $. 1.41 MUTUALLY EXCLUSIVE EVENTS Two events E,, E, sample points in common i.e., E, 0 of a sample space S are said to be mutually exclusive if they have no > Mutually exclusive events are sometimes called as Disjoint Events. 4.12. COMPLEMENTARY EVENTS ‘Two events of a sample space whose intersection is @ and whose union is the entire sample space are called complementary events. Thus if F is an event of a sample space S, its complement is denoted by £" or E and EQ E =$,EU E=S,E = S-E. 1.43 PROBABILITY-AXIOMATIC APPROACH Definition : Let Sbe a finite sample space. A real valued function P from the power set of S into R is called a probability function on S if the following three axioms are satisfied. Axioms of Probability : [JNTU (A) 2010 S, 09 (Set No. 3), (H) 09, Dec. 14] P(E) > 0 for every subset E of S. (9 Axiom of positivit (ii) Axiom of certainity : P(S) = 1 (iii) Axiom of union : If E,, £, are disjoint subsets of S, then PE, UE) = PE) + PE) ‘The image P(E) of E is called the Probability of the Event E. Note : If E,, Ey... E, are disjoint subsets of S, then P(E, U By U wed E,) = P(E,) + P(E) + svect PCE.) This is the generalisation of axiom (i). 1.14 PROBABILITY OF AN EVENT INA SAMPLE SPACE CONTAINING EQUALLY LIKELY SIMPLE EVENTS Let a sample space S consists of n equally likely simple events E,, E, 2 PE) = P(E) = = PE) Since $= E,W Ey UU Ey we have P(S) = P(E) + P(E,) + + PCE,) 1 1 = PE) + PE) + +P) he =m PE) = PE) = Hence P(E,) = P(E,) Let an event BS be such that B= E,W By 0. U Ey m are two events, then @ PCE, OE) = P(E) ~ P(E, VE) (6) PE, Ey) =1- P(E, VE) (0) P(E, 0B) =1- P(E Ey) Cor 3. If ,,£ and E3 are three events, then P(E V Ez V Es) = P(E\) + P(E) + P(E) ~ P(E, 0 Ez) P(E, OE3) =P(E3 OE) + PCE OE, OE) A card is drawn from a well shuffled pack of cards. What is the probability that it is either a spade or an ace ? [INTU Nov. 2003S (Set No. 2)] Solution : Let S'is the sample space of all the simple events. 2. m(s) = 52 Let A denote the event of getting a spade and B denote the event of getting an ace. Then A U B= The event of getting a spade or an ace ACB = The event of getting a spade and an ace 2 pw- 4, pansy- + 52° PB) = 5 ANB) = 55 . PA By Addition Theorem, P(AUB) = P(A) + P(B) - PAB) 134 _1_16 52° 52° 52 «5213 Three students A, B, C are in running race. A and B have the same probability of winning and each is twice as likely to win as C. Find the probability that B or C wins. [JNTU April 2006, (A) Nov. 2010 (Set No. 4)] Solution : AU BUC = = Sample space of race Bydata, P(A) = P(B) and P(A) = 2 P(C) eM) Wehave P(A) + P(B) + P(C)= 1 > 2P(C) +2 P(E) + P(C)= 1 [by (1)] 1 2 2 => PC)= 5. PUA) = 3 and P(B) = 5 Review of Probability 15 The probability that B or C wins = P(BUC) win a 5 From a city 3 news papers A, B, C are being published. A is read by 20%, Bis read by 16%, C is ready by 14% both A and B are read by 8%, both A and C are read by 5% both Band C are read by 4% and all three A, B, C are read by 2%. What is the percentage of the population that read at least one paper. = P(B) + P(C)- PBAO) 4 Solution : Given P(A) = foe P(B) = co »PCO= 799 and Puns)= ©, pang =, BAG = A and PUNBO = 100° i 100 100 P(ALBUO) = P(A) + PCB) + PCC) = PAB) = P(BAC) = PANO) + PUOBAO) 20 16 i 8 4 5 2 35 ~ 100 * 100 * 100 ~ 100 ~ 100 ~ 100 * 100 ~ 100 35, Percentage of the population that read at least one paper = 755 * 100= 35 4.16 CONDITIONAL EVENT Def : If E,, £, are events of a sample space S and if E, occurs after the occurrence of E,, then the event of occurrence of E, after the event £, is called conditional event of £, given E; E E,, Wis denoted by |" (or Ey/E,). Similarly, we define (or E,/ E). 1 2 eg. 1. Two coins are tossed. The event of getting two tails given that there is atleast one tail is a conditional event. eg. 2. Two unbiased dice are thrown. If the sum of the numbers thrown on them is 7, the event of getting 1 on anyone of them is a conditional event. eg. 3. A die is thrown 3 times. The event of getting the sum of the numbers thrown is 15 when it is known that the first throw was a 5 is a conditional event. 4.17 CONDITIONAL PROBABILITY Def : If E, and E, are two events in a sample space S and P(E,) + 0, then the probability of , after the event £, has occured, is called the conditional probability of the event of E, given PC Ey Ey E, and is denoted by A= 2) or P(E, /E,) and we define {& Ra } 4 4) PULA) Similarly, we define P * (é PCE) 16 Probably and Statistics & Complex Variables Wehave PE) = PCE) ~~ n(E,)/m(S)~ (E,) Number of elements in Ey 0 Ey Number of elements in E, Here £, ¢ Sand P(E,) > 0. We agreed to consider only those elements (sample points) of the event E, as the sample space and £, to be another subset of S. Since E, is the new sample space, the only elements of E, that concer us are also the elements of E, i.e., the elements of E, 0 E,. Then we define the probability of B, relative to the new sample space E, ie., the conditional probability of E, given E,. Thus some authors define the conditional probability of E, given E, p( £2) gg MOE) te, P |e | as nae 1.18 IF A,B, CARE THREE EVENTS OF A SAMPLE SPACE WE HAVE SOME USEFUL RESULTS (4) _ P(ADA) _ P(A) aS PCA) = P(A) = (P(A) > 9) 0 P(A) 7 P(AY 7 Pcay 7 (P(A) > 0) (8) _ POA) _ PO) 3. A-B=A~ (ANB) = P(A ~ B) = P(A ~ (AB) => P(A -B)= P(A) P(AMB) [2 ABCA] 4.40 B= ANCCBAC = PIANC) < P(BNC) PIANC) . PBOO) (4 2) 7 =P = MO =rla) 0 5. A, B are mutually exclusive events and P(B) > 0 P(ADB) 0 PB) PB) ° 4 => P(ANB) = 0 and P(B)>0 results (4) = 6. P(AMB) = 0 and P(B) # 1 A P(ANB) = P(A=(ANB)_ P(A)- P(ANB) P(A)-0 > AF] = pm = pa) = =P) = «T= PB) PIB) P(B) ~ I= P(B) ~ 1 P(B) A) _PUA) P(B) Review of Probability 7 7. P(A) + P(B) = P(AMB) = P(AUB) = P(A) + P(B) ~ P(AMB) <1 Plans), RA+P(B)=1 = PA) + PB) ~ 1s PNB) = TED 2 TG) (4) P(A)+ P(B)=1 =a) 2 PB) A 8. (4) . P(B) + (4) » P(B) = P(A B) + P(A B) = P(AMB) + P(A - (AMB)) = P(AMB) + P(A) ~ P(AMB) = P(A) A A) V5 + Pla} PB) + (4) P(B) = P(A) 4A P(ANB) = P(S-(AUB)-P(S)- (AUB) 1- P(AU B) ae B)~ PB) ~ 1-PB) ~~ I-P(B) ~ ‘I1-P(B) 10. A and B are mutually exclusive and P(AUB) #0 => P(AMB) = 0 and P(AUB) #0. ( A ) P(AN(AVB)) P(A) P(A) NowP (4B) ~~ P(AGB) ~~ P(A)+P(B)- (ANB) P(A) + PCB) 11, B, Care mutually exclusive events = —P(BNC) = 0 Buc P(BUC)N A) P(BOA)U(COA) Now P|] PAY P(A) P(BO A)+ P(COA) - P(A) (cr BAA, COA are mutually exclusive) P(BO A) P(COA) PA) * P(A) 2 (759) --(i) +3) 12, For any three events A, B and C oA w» fee 8-8) Proof : (i) We have P(AUB) = P(A) + P(B)-P(AMB) Probability and Statistics & Complex Variables Replace A and B by the compound events AC and BAC respectively. P(ANQUBAC)]= Hence PUAYB)OC] P(C) (4? Cc eg. 1: Let a die be rolled and A = {1, 3, 5}, B= PUNOUBAC) _ PANC) , BAC) _PANBOC) PC) co(d)n(Bh 228) Jefe w (seb ol $28) P(ANC)+P(BAC)-P(ANBOC) »P(C) #0 PC) PC) P(AQBAC) , PANBAC) be the events associated with the random experiment. Sol. Now P(1) 4)- 43)- i MA ral c £ a P| ()- e)- (@)- (¢)- 2 c P(BOA) P(A) PCAOB) PCB) P(COA) P(A) PANO) PC) P(COB) P(B) PBC) P(C) PO PO = ManBndysrdosnd , M4ae),. »( 4) - PO) “PO AC. {2, 3}and C = {2, 3, 4,5} 1 _ Pu) © P35) TTT 3 6°66 a POY) =e © PC2,33) ~ Lt 2 66 ee PU3,5)) 66 2 ~ P35) ~ TTT 7 3 666 1 “2 P23) _ P({2,3}) 11 = P23) _ 66 ao ~ P2345) ~ T1172 6°66 6 Review of Probability 19 1 qr then find 1 eg. 2: If P(AY) = a P(BS) = 5 and P(ANB) ae - @ (4) and (4) (ii) [2] and [2-} Sol. P(B) = 1 - PB) = 1 - - ; and P(AMB) = ; A) PUANB) 4 ‘B) PBA) 1/4 2 © PB) pay ~ 172 > 2 EP) PA) > 5785 AL) PAS OBS) _ _ Le P(4) + P(B)~ P(A) GO PUB) pceey = ~ PBS) [et-2] g'2 4} 1 T “4 2 1,3 1 Bo)_ PBS OAS) PBA) | 2°84) 1 Now PUT} PCA) PAS) 3 “3 4.49 MULTIPLICATION THEOREM OF PROBABILITY a Statement : In a random experiment if E,, E, are two events such that P(E,)#0 and P(E,)#0, then PUE,ME,) = PE,) « PUEYE,) 5 (EM E,) = PE,) - P(E,/E,) Proof: Let S be the sample space associated with the random experiment. Let E,, E, be two events of S such that P(E,) + 0, P(E,) # 0. Since P(E,) 4 0, by the definition of conditional probability of £, given E,, P(E OE) P(E) P(E, |B) = PE, OE) = PE) 2} P(E, OE) Since P(E,) # 0, we have P(E;|E>) = pep) P(Ey) Hence P(E,OE,) = P(E,).P(E, | E2) 1 1 4) PLANB) _ 1/6 1 pay= 4, then of 2 aria eg. 1. If (ANB) 1 P(A) 1/23 eg. 2. If A, B are two events (#4) then P(A UB) = IP (498) = — P(A). P(BIA) 20 Probabilly and Statistics & Complex Variables Note : Multiplication theorem can be extended three events E,, By, E, as : PE, QE,OE,) = P(E, 0 £2) 0 E3) = P(E, 0 Ey). P(E3 | Ey OE) = P(E,). P(E2 | Ey) P(E3| Ey 0 Ea) This result can be extended to 4 or more events. 4.20 COMPOUND EVENT Definition : When two or more events occur in conjunction with each other, their joint ‘occurence is called compound event. eg. 1. If balls are drawn froma bag containing 4 green, 6 black, 7 white balls, the event of drawing 2 green balls or 2 white balls is a compound event. eg. 2. Whenadie and a coin are tossed the event of getting utmost 4 on the die and head on the coin is a compound event and separately they are independent events. eg. 3. From a pack of cards if a spade card is drawn and not replaced and if again a club card is drawn, the two drawings form a compound event and separately they are dependent events. Note : Multiplication theorem of probability is also called theorem of compound probability 4.21 INDEPENDENT AND DEPENDENT EVENTS If the occurrence of the event £, is not affected by the occurrence or non-occurrence of E the event £,, then the event E, is said to be independent of E, and {2 = PE). Hl If P(E,) # 0, P(E,) 4 0 and E, is independent of E,, then E, is independent of £,. In this case we say that E,, E, are mutually indpendent or simply independent. If the occurrence of the event £, is effected by the occurrence of £,, then the events E,, E, E, are dependent and {2} + PE). f Note :1. The event E is independent of @ and S. 2. IfA and B are independent events ofa sample space S, then (4, B),(A, B) and (4, B) are also independent. 1.21.1 PAIRWISE INDEPENDENT EVENTS: Three events £,,, and £, are mutually independent or simply independent, if (P(E, OE) = P(E\). PE), P(Ex 0 E3) = P(E2).P(E3)» P(E 0 Ej) = P(E3)-P(E\), (i) PE, OB, Es) = P(E) P(E2)- P(E3) Note : Probability of occurrence of at least one of the independent events E Ey, E3 = P(By VU Ey VE3) Review of Probability a Find the probability of drawing 2 red balls in succession from a bag containing 4 red and 5 black balls when the ball that is drawn first is (i) not replaced (ii) replaced. Solution: Let £, be the event of drawing a red ball in the first draw and B, be the event of drawing a red ball in second draw also. (i After the first draw the ball is not replaced. The first ball can be drawn in 9 ways and the second in 8 ways since the first ball is not replaced. Then both the balls can be drawn in 9x 8 ways, There are 4 ways in which £, can occur and 3 ways in which £, can occur, so that E, and E, can occur in 4 x 3 ways. E (2) = P(E, given the probability of E,) 1 = P(2nd ball is red, given that first ball is red) = E,) 4 3 1 [ » PEA E,) = PE) * |= 2 x 2-4 | Pe, o PEO E,) = P(E,) rt &) 9*376 (A) (ii) Suppose the ball is replaced after the first draw. Then pene) =<. 2-8 OE) 9-9 Bt [ERMINE A class has 10 boys and 5 girls. Three students are selected at random one after another. Find the probability that (i) first two are boys and third is girl (i/) First and third are of same sex and the second is of opposite sex. [JNTU 2004S, Feb. 2008S (Set No. 3)] Solution: Total no. of students = 10 +5 = 15 (The probability that first two are boys and the third is girl is ragagy~%.2 515 PEOEOF) ~ 15-14-13 91 (i) Suppose the first and third are boys and second is a girl 10 5 Probability of the event = PUE,) = 75. 7g Suppose first and third are girls and second is boy. 5 10 4 20 5° 14°13 273 Then the probability of the event = P(E,) Required probability = P(E,) + P(E;) 15, 20 45420 65 “or 273 273 273 88 22 Probability and Statistics & Complex Variables Two aeroplanes bomb a target in succession. The probability of each correctly scoring a hit is 0.3 and 0.2 respectively. The second will bomb only if the first misses the target. Find the probability that (i) target is hit (ii) both fails to score hits. [INTU 20048 (Set No. 3)] Solution : Let A be the event of first plane hitting the target and B be the event of 2nd plane hitting the target The probability of Ist plane hitting the target = P(4) = 0.3 The probability of 2nd plane hitting the target = P(B) = 0.2 P(A) P(A) = 0.7 and P(B) = 1 — P(B) = 0.8 (@ _ P(target is hit) = P{(A hits) (or) (A fails and B hits)] = PALA B)] = P(A) + P( 4.08) By Addition Theorem) = P(A) + P(A). PB) (By Multiplication theorem, since 4 and B are independent) = 0.3 + (0.7) (0.2) = 0.44 (ii) P(both fails) = P(A fails and B fails) = P(AMB) = P(A) - PCB) = (0.7) (0.8) = 0.56 B A Determine (i) (2) 0) (4) if A and B are events with 1 P(A)=3, =5 [JNTU Dec. 20045 (Set No. 2)] Solution: Given P(A) = i PCB) = i P(AUB) =4 Now P(AnB) = P(A) + P(B) - P(A U B) Lliila ~3°4°2° 2 . (2) _ PUAOB) 12 | @O PA)= Pa = 1 = ii 6 13 (ii) PY) =1-PB)=1- 4-9 P(ANBY) = P(A) - PUANB) = 3 - 73 = P(BS) 3/4 3 Review of Probability 23 Box A contains 5 red and 3 white marbles and box B contains 2 red and 6 white marbles. If'a marble is drawn from each box, what is the probability that they are both of same colour. [NTU May 2006 (Set No. 3), Aug 2007, (A) Nov. 2010 (Set No. 2)] Solution : Suppose £, = The event that the marble is drawn from box A and is red. PE) 3 The probability that both the marbles are red is EAE) = PE). PE) =~ b= PE,ME) = PE) - PE) = 7g + = Tag Let E, = The event that the marble drawn from box A and is white 3 16 Let E, = The event of marble drawn from box B is white 163 PED = 7-9-5 d PEE) = 3 =o and PUEOE) = 76 - 3 ~ 128 The probability that the marbles are of same colour = P(E, E,) + P(E,OE,) 5 9 14 7 =, 2M _ 7 orgy 128 * 128 ~ 128 ~ 64 ‘Two marbles are drawn in succession from a box containing 10 red, 30 white, 20 blue and 15 orange marbles, with replacement being made after each draw. Find the probability that ( Both are white (i) First is red and second is white [SNTU 2005S, 2006, 2007, Nov. 2008, 2008S, (A) Nov. 2010 (Set No.1)] Solution: Total no. of marbles in the box = 75 () Let E,be the event of the first drawn marble is white. Then 30 PE) = 5 Let E, be the event of second drawn marble is also white. Then 30 P(E,) = 5 24 Probability and Statistics & Complex Variables The probability that both marbles are white (with replacement) 30 30 4 = PEE, = PE). P(ExIE) = 55 + 35 = 35 (ii) Let £, be the event that the first drawn marble is red. Then 10 2 PE) = 95 = 75 Let E, be the event that the drawn marble is white. Then 302 P(E|Ei)= 35 = 5 -. The probability that the First marble is red and Second marble is white = PEE,OE,) = P(E) - P(Ey | Fi) 22.4 15° S75 Three boxes, practically indistinguishable in appearance have two drawers each. Box | contains a gold coin in first and silver coin in the other drawer, Box 2 contains a ‘gold coin in each drawer and Box 3 contains a silver coin in each drawer. One box is chosen at random and one of its drawers is opened at random and a gold coin is found, What is the probability that the other drawer contains a coin of silver. [JNTU 2004 (Set No.2)] Solution: Let £, denote the event that the box is choosen, i = 1, 2, 3. 1 PE) = 3 fori=1,2,3 Let A be the event that the gold coin is chosen. Then P(A|E,) = Probabiity that a gold coin is choosen from the box i= 1, 2, 3 P(A|E,)= 5 (+ The total no. of coins in box 1 is 2) 1 2 2 P(A|E,)= 5 = 1. (There are two gold coins in box 2) 0 and P(A|E3) (There is no gold coin in box 3) (The probability that the drawn coin is gold P(A) =P(E,) P(A EB) + P(E,) P(A| Ea) + PCE.) P(A| ES) 1 2 Review of Probability 1 ies that students A, B, C, D solve a problem are =, Pranic 1 4 respectively. Ifall of them try to solve the problem, what is the probability that the problem is solved, INTU 2007 (Set No. 4)] Solution : Given the probability of 4, B, C, D solving the problem is 1 2 1 1 P(A) = 35 PB) = 5 PO = 5s PO) = The probability that the problem is not solved by A, B, C, D are 2 a 3 pa _4 P(A) The probability that the problem is not solved when 4, B, C, D try together (independently) = P(AQABOCND) = P(A) . P(B) . P(E) . P(D) 234 3_ 6 3°5°5°4° 25 £2 25° 25 Two digits are selected at random from the digits 1 through 9. (0 If the sum is odd, what is the probability that 2 is one of the numbers selected ? (i) If2 is one of the digits selected, what is the probability that the sum is odd ? [JNTU 2005, 2006S (Set No. 1)] Solution: The given set consists of five odd digits (1, 3, 5,7, 9) and four even digits (2, 4, 6, 8). We know that . The probability that the problem is solved = even + even = even even + odd = odd odd + even = odd odd + odd = even x4=20 (i) Total number of events = If ‘2’ is one of the digits, then the other digit must be odd. Number of ways = 5 sd So, required probability = = = + (ii) 1f2 is selected, then the remaining number of digits = +. Total events = 8 26 Probability and Statistics & Complex Variables If2 is one of the digits selected, the probability that the sum is odd Favourable cases forodd 5 - Total events o8 A,B, C, are aiming to shoot a balloon. A will succeed 4 times out of 5 attempts. The chance of B to shoot the balloon is 3 out of 4 and that of C is 2 out of 3. If the three aim the balloon simultaneously, then find the probability that at least two of them hit the balloon. [NTU Nov. 2008, (H) Dec. 2011 (Set No. 1)] Solution: The probability of A hitting the target = P(A) = The probability of B hitting the target = P(B) = 3 The probability of C hitting the target = P(C) a The probabilities of A, B, C not hitting the target respectively are Now the probability that exactly two will hit the balloon = P(ANBAC)+P(ANBAC)+P(ANBNC) = P(A). P(B) . P(C) + P(A) . P(B) . P(C) + P(A) . P(B) . P(C) 431,412,132 54355435543 124846 26 13 ~ 60 30 The probability that all will hit the balloon = P(A ~BOC) . 432 = P(A). PB). PC)= 5-55 .. The probability that at least two of them will hit the target _ 13213412 _ 255 30°5 3030 PRUMTINNIY There are 12 cards numbered | to 12 in a box, if two cards are selected what is the probability that sum is odd i) With replacement ? ii) Without replacement ? [JNTU Apr. 2009 (Set No. 2)] Solution: The cards having even number = {2, 4, 6, 8, 10, 12} The cards with odd numbres = {1, 3, 5,7, 9, 11} Review of Probability With replacement : Suppose we select a card place it back and select one card. This can happen in 12.12 = 144 ways. To have an odd sum, (0 the first card has to be odd and the second has to be even or (ii) the first card has to be even and the second has to be odd. Number of these favourable ways = 6.6 + 6.6 = 72 Rid ~ P(sum is odd) = 775 = 5 Without replacement : The total number of outcomes = (12)(11) = 132 ‘The sum can be odd as below : (i) The card in first draw is even and on the second is odd. 6 6 36 Probability (for )= 75-477 739 (ii) The card in first draw is odd and on the second is even. 66 36 Pfor GD) = I-41 ~ 132 P(ih odd) = 26.426 2 6 ~ Pathe sum is odd) = 739 * 132 ~ 132 ~ 11 E nm Three machines I, II, III produce 40%, 30%, 30% of the total number of items of factory. The percentages of defective items of these machines are 4%, 2%, 3%. If an item is selected at random, find the probability that the item is defective. [INTU Apr. 2009 (Set No. 2)] Solution: Let A, B and C be the events that the machines I, Il and III be chosen respectively and let D be the event which denotes the defective item. 30 Fo9 7 030. 30 9 7 030, PO 40. 9 = 0:40, PB) = 755 100 From the given data, we have (2)- A - 004 o(B)- a = 002,7(2) - = 003 A) 100 *\B)~ 100 Cc} 100 The probability that the selected item at random is defective is Given that P(A) = e-ave(2) +rom7(2) eneae(2) A B c 404 30 2 30 3 164649 ~ 700 * 100 * 100 * 100 * 100 * 100 ~~ 1000 1000 [RRINIEER] 10 dice are thrown. Let A be the event that the sum of the points on the faces is 9. Let B be the event that at least one number is 6. Find () P(AMB) (ii) (AUB) (iii) P(AS UBS). INTU Apr. 2009 (Set No.3), (H) May 2012 (Set No. 4)] 28 Probability and Statistics & Complex Variables s There are 36 simple outcomes when two dice are thrown, The event A(= that a sum 9) occurs in the following way A = {B, 6) (4, 5), (5,4), (63) 4 PIA)= 36 The event B that at least one number is 6 occurs in the following way B = {(6,1), (6,2), (6,3), (6,4), (6,5), (6.6), (1,6), (2,6), (3,6), (4,6), (5.6) } 5 u PB) = 36 Now ANB = {(3,6), (6,3)} 2041 1) PAB) = 36 = Tg (A) + PCB) - (ANB) 4 u 2_33 ii) P(AUB) =sta-= 36-36-36 (36 fighting for the claim of Chief Minister ship. The chances of winning the election of the 3 parties are in the ratio 1 :2 : 3 respectively. The probability that the party X if selected, will introduce total arrack prohibition in that state is 1/2. The probability that the party Y if selected, will introduce total arrack prohibition in the state is 1/4 and the prohibition that the party Z if selected, will introduce total arrack prohibition in the state is 3/4. What is the probability that there will be a total prohibition in the state after the election in year 2002. [INTU Apr. 2009 (Set No.3)] Solution : Let P(X wins) = P(X) PCY wins) = P(Y) and P(Z wins) = P(Z) Since P(X): PCY): P(@Z):: 1:2:3 1 2 3 P(X)= = 5 PLY) = 5 PIZ)= = = — POY) sPQD=¢ Let A be the event of introducting total arrack prohibition. It is given that 1 1 3 P(AIX) = 5 3 POY) = 73 PAZ) = 5 P(A) = PAIX) . P(X) + P(A/Y).P(Y) + P(A/Z)-P(Z) Li 12,33 ep.ctz.ctsee 2°6 4°6 46 eo eae “12° 12" 247 24 Review of Probability 29 Ina group consisting of equal number men and women, 10% of the men and 45% of the women are unemployed. If a person is selected randomly from the group then find the probability that the person is an employee. [INTU(A) Dec. 2009 (Set No. 3)] 1 1 3° P (Women) = POY) = Let A be the event of employed and E be the unemployed event. then (4) 1 pf £).8 M/ 100 10 M/ 10 of @)-85 -48, of) 85 ion“ 10°"r) “0 .. Probability that the person is employed = P(M). rE) PW), () 1 ( 9 5. 5) 145 =1/2 32) 8 240 10 20 If the probability that a communication system will have high fidelity is 0.81 and the probability that it will have high fidelity and selectiveity is 0.18. What is the probability that a system with high fidelity will also have high selectivity ? [JNTU (A) Dec. 2009 (Set No. 4)] Solution : Let F be the event of communication system will have high fidelity and S be the event of communication system will have high fidelity and selectivity. Then P(F)=0.81 and PFS) =0.18 :, The probability that a system with high fidelity will have high selectivity also =°(8)-P0h) 0.18 _2 Se ion : We have P (Men) = P(M) 728 F) PF) 0819 [BRIIEERD 4 fair dic is tossed twice. Find the probability of getting a 4,5 or 6 on the first toss and 1,2,3 or 4 on the second toss. [SNTU (K) 2009 (Set No.1)] Solution : Let E be the required event. Total number of sample points = 6x6 =36 The favourable sample points are (4,1),(4,2),(4,3).(4.4) 5 (5.1)4(5,2)5(5,3)5(5,4) 5 (6,1), (6,2),(6.3),(6.4) probability that they are both aces if the first card is (i) replaced (ii) not replaced. [INTU (K) 2009 (Set No. 2)] We understand that 2 cards are drawn one after another. Solutic 30 Probability and Statistics & Complex Variables Let E be the event that the two cards are aces. (When the card drawn first is replaced, we have n (S) = 52x52 ee 52°52 13x13 169 (ii) When the card drawn first is not replaced, we have 4301 32°51 x17 22 SEXERI The probability that India wins a cricket test match against West Indies is known to be 2/5. If India and West Indies play three test matches, what is the probability that (i) India will lose all the three matches (ii) India will win at least one test match (iii) India will win at most one match ? [JNTU (K) 2009 (Set No. 4)] Solution : Given that probability of India winning against West - Indies = PW) = 3 Probability of losing against West - Indies = PO?) => (i) The Probability that India will lose all the three matches 33,327 3°5°5 125 (i) The Probability that India will win at least one test match = PW) x PW) PW) 2798 125° 125 =1~Probability that India will win none of the test matches = (iii) The Probability that India will win at most one match = The Probability that India will win one match or no matches = PW). PW). PW) + PW). POF). PW) mee “S55 55'S 125 25 1.22, BAYE’S THEOREM [JNTU 2007S, 2008, (A), (If) Dec. 2009, (A) Nov. 2010 (Set No. 4] EEE, ate n mutually exclusive and exhaustive events such that P(E) > 0 (= 1, 2ynony m) in a sample space S and A is any other event in S intersecting with every E, (i.e., A can only occur in combination with any one of the events E,, E,,....E,) such that P(A) > 0. If E, is any of the events of E,, Ey. where P(E,), P(E,)y-sP(E,) and P(A/E,), P(AJ Ey) ysP(A/E,) are known, then . P(E). P(AI Ex) PU!) ~ BCE, PATE) + PE). P(AI Ey) +o PCE,) (AT E,) Review of Probability 31 Proof: E,, EE, are n events of S such that P(E) > 0 and E, 0 E,= 6 for i +/ where UJ =U, Yom Me AlSO Ey, EyyunsE,, are exhaustive events of S and A is any other event of S where P(A) > 0. S= B,UE,U.WUE, and A= AOS = ANE UE UE) = (ANE, VANE) (ANE,) Here ANE), AME, ny AOE, are mutually exclusive events. Then P(EK VA) P(E, OA) PEKRIAV= Dg) = PARE) U(AN BU. U ADB, _ PEK OA) ~ PAN E\) + PAA Ep) +. + PANE, ) P(Ex)-P(A/ Ex) * P(E,).P(AT E,).+ P(E) P(A Ey) +0 P(E,)-P(ATE,) Note : Baye’s theorem is also known as formula for the Probability of “Causes”, ic., probability of a particular (cause) E, given that event A has happened (already). P(E,) is ‘a priori probability’ known even before the experiment, P(4/E,) “Likelihoods” and P(E,/ A) ‘Posteriori Probabilities’ determined after the result of the SOLVED EXAMPL Inacertain college, 25% of boys and 10% of girls are studying mathematics. The girls constitute 60% of the student body. (a) What is the probability that mathematics is being studied ? (6) If a student is selected at random and is found to be studying mathematics, experiment. find the probability that the student is a girl? (c) a boy ? [JNTU 2004 (Set No. 2)] Solution : Given P (Boy) = P(B)= 22. = 2 olution : Given P (Boy) = P(B)= 799 = 5 d P (Girl) = P(G) oa and P (Girl) = P(G) = Tog = 5 Probability that mathematics is studied given that the student is a boy = P(M / B) Probability that mathematics is studied given that the student is a girl = P(M/G) (a) Probability that the student studied Mathematics = P(M) = PG) P(MIG) + P(B) P(MIB) 32 Probability and Statistics & Complex Variables By total probability theorem, -3 1,2 14 POD~ 35-10 5° 4 25 (6) By Baye’s theorem, probability of mathematics student is a girl = P(G/M) 31 P(G)P(M/G) _ 519 3 PM) 88 2 (c) Probability of maths student is a boy = P(B/M) 2 5 lni— _ P(B) P(M/B) “ P(M) The chance that doctor A will diagnose a disease x correctly is 60%. The chance that a patient will die by his treatment after correct diagnosis is 40% and the chance of death by wrong diagnosis is 70%. A patient of doctor A, who had disease x, died. What is the chance that his disease was diagnised correctly. Solution: Let £, be the event that “disease x is diagnosed correctly by doctor A” and E, be the event that “a patient of doctor A who has disease x died”. 60 40 Then P(E) = 799 = 0.6, P(E E,) = 79g = 04 70 Now P(E,)=1-0.6 = 0.4 and P(E,/E,) = 7 = 0.7 :. By Baye’s theorem, 100 ROPE PU) P(Ea/ Ey) 102) PE) PE /Ex)+PE)-PE2/ Ey _ 06x04 6 © 0.6x0.44+0.4x0.7 13 |A bag A contains 2 white and 3 red balls and a bag B contains 4 white and 5 red balls. One ball is drawn at random from one of the bags and it is found to be red. Find the probability that the red ball drawn is from bag B. [JNTU (A) Nov. 2011 (Set No. 4)] Solution: Let 4 and B denote the events of selecting bag A and bag B respectively. 1 1 Then P(4) = 5, PUB) = >- Let R denote the event of drawing a red ball. Review of Probability Having selected bag A, the probability to draw a red ball from A= P(R/ A) = = elu Similarly, P(R/ A) = ‘One of the bags is selected at random and from it a ball is drawn at random. It is found to be red. Then the probability that the selected bag is B P(B).P(R/B) P(A).P(RI A) + P(B)P(RTB) 1 black, | red, 2 white balls and third box contains 5 black, 3 red, 4 white balls. Of these a box is selected at random. From it a red ball is randomly drawn. If the ball is red, find the probability that it is from second box. Solution : Let x, y, z be the first, second and third boxes 1 1 1 PE) = 5. PO)= 3 PED= 5 Let R be the event of drawing a red ball from a box. 3 1 3 So P(R/x) = §, P(R/y) = 4, P(R/2) = 55 By Baye’s theorem, the required probability PCy). P(R/ y) POIR) = “BG}(RTx) + PQ). PUR] 9) + P@)-P(R/ 2) 1 iit 3 fi 3.693 "4312 ‘ Suppose 5 men out of 100 and 25 women out of 10,000 are colour blind. A colour blind person is choosen at random. What is the probability of the person being a male (Assume male and female to be in equal numbers) ? [SNTU April 2004,(A) Nov. 2010 (Set No.2)] Solution : Given that 5 men out of 100 and 25 women out of 10000 are color blind. A colour blind person is choosen at random. 1 The probability that the choosen person is male = P(M) = > Similarly the probability that the choosen person is female = PU) = 5 34 Probability and Statistics & Complex Variables Let B represent a blind person. Then % P(BIM) = 7pq = 0.05 25 1 P(BIW) = To000 = 400 = 925 The probability that the choosen person is male is given by P(BIM).P(M) P(M!B) = BoM). P(BIM) + PW). (BIW) 0.05% 0.5 ~ (.05x.5) + (.5x.0025) Ina bolt factory machines A, B, C manufacture 20%, 30% and 50% of the total of their output and 6%, 3% and 2% are defective. A bolt is drawn at random and found to be defective. Find the probabilities that it is manufactured from (i) Machine A (ii) Machine B (iii) Machine C, [JNTU 2007 (Set No. 4), (A) Apr. 2012 (Set No. 2)] Solution: Let P(A), P(B), P(C) be the probabilities of the events that the bolts are manufactured by the machines 4, B, C respectively. Then 20 1 yp 30 3 501 “100 3°) F099 ~ 10° > G00 ~ 2 Let D denote that the bolt is defective. Then 0.95 6 P(DI A)= 795. P(D!B) = FOG» 00 () If bolt is defective, then the probability that it is from machine A = P(A/ D) P(DIC) = P(D/ A). P(A) 12 P(D/ A). P(A)+ P(D/B). P(B)+ P(D/C). PC) ~ 31 Similarly, (i?) P(B/D) = 2 (ii) P(CID) = 5 Of the three men, the chances that a politician, a business man or an academician will be appointed as a vice-chancellor (V.C) of a University are 0.5, 0.3, 0.2 respectively. Probability that research is promoted by these persons if they are appointed as V.C are 0.3, 0.7, 0.8 respectively. (® Determine the probability that research is promoted (ii) If research is promoted, what is the probability that V.C is an academician? [JNTU 2007, 2007S, 2008S (Set No. 1), (HH) Dec. 2013] Solution : Let 4, B, Cbe the events that a politician, businessmen or an academician will be appointed as V.C of the three men. Review of Probability 35 Then P(A) =0.5, P(B) = 0.3, P(C) = 0.2 The probabilities that research is promoted if they are appointed as V.Cs are P(RIA) = 0.3, P(RIB) = 0.7, P(RIC) = 0.8 (The probability that the research is promoted = P(A).P(RIA) + P(B).P(RIB) + P(C).P(RIC) (0.5)(0.3) + (0.3)(0.7) + (0.2)(0.8) (ii) The probability that research is promoted when the V.C is an academician 0.52 _ P(C).P(R/C) me) P(C). P(R/ C) + P(B). P(R/ B) + P(A). P(R/ A) 0.16 4 ~ Daseo2tsore © 13 ~ 0307 A businessman goes to hotels X,Y, Z, 20%, 50%, 30% of the time respectively. It is known that 5%, 4%, 8% of the rooms in X, Y, Z hotels have faulty plumbings. Whaat is the probability that business man’s room having faulty plumbing is assigned to hotel Z? [JNTU 2007, 2008 , (K) 2009, (A) Nov. 2010 (Set No. 3) | Solution : Let the probabilities of business man going to hotels X, Y, Z be respectively P(X), P(X), P(Z). Then 202 50S. 30 3 PO = F099 = 10% PO Joo = 10! too ~ 10 Let £ be the event that the hotel room has faulty plumbing. Then the probabilities that hotels X, ¥, Zhave faulty plumbing are Pex) => = 4; pein = + = 1 pe@iy-% -2 100 ~ 20 100 ~ 25 100 ~ 25 The probability that the business man’s room having faulty plumbing is assigned to hotel Z P(Z). P(E IZ) P(Z). P(E Z)+ P(Y). P(E Y)+ P(X). P(E/X) 8 2 P(ZIE) = a 32,351,271 9 10°25" 10°25" 10°20 FIRE [2 2 factory, machine A produces 40% of the output and machine B produces 60%. On the average, 9 items in 1000 produced by A are defective and 1 item in 250 produced by B is defective. An item drawn at random from a day’s output is defective. What is the probability that it was produced by A or B ? [INTU 20048, (A) Nov. 2010 (Set No. 1)] Solution : Output produced by A = 40% P(A) = 0.4 36 Probability and Statistics & Complex Variables Output produced by B = 60% 2. P(B)= 0.6 9 P(D1 A) = Probability that items produced by A are defective = +555 i ae Similarly, P(D/ B) = 555 = 0.004 P(A/D) = Probability that the bolt is produced by A given that itis defective P(A)xP(D/ A) P(A)x P(D/ A)+ P(B)x P(DTB) 0.4 x 0.009 ~ 0.4 x 0.009 + 0.6 0.004 0036 0.0036 + 0.0024 P(B/D) = Probability of manufacturing the defective bolt by machine B 0.0024 006 0.006 (or) Since P(A/D) = 0.6, therefore, P(B/ D) = 0.4 Note : Strictly speaking, the probability that it is produced by A or B = 04 1. ‘A manufacturer firm produces steel pipes in three plants, with daily production volume of 500; 1000 and 2000 units respectively. According to past experience; it is known that the fraction of defective outputs produced by these plants are respectively 0.005; 0.008 and 0.010. If pipe is selected from day’s total production and found to be defective, find out (i) from which plant the pipe came (ii) what is the probability that it came from the first plant ? Solution : Let 4,,4p, 4s be the events that a pipe is manufactured in plant 1, 2 or 3 respectively 500 1 1000 _2 2000 _ 4 » P(A)= 2 =, Ply) = P(A) = (A= 3509 = 77 42)= 3509-7 4)= 3500 7° Let B be the event that a defective pipe is drawn Then P(B| 4,)=0.005, P (B| 4) = 0.008, P (B| 43) = 0.010 Baye’s theorem gives (4)- P(B/ A). P(A) B P(B/ A). P(A\)+ P(B/ Ap). P(Ay)+ P(B/ Az). P(A3) 0.005 7 (eons-2)s[ooos2)[onioxd) 8 7. 7. Review of Probability 37 0.016/7 _16 pcg, / py = 9040/7 _ 40 0061/7 61 061/761 Thus it is most probable that the defective pipe has been drawn from the output of the third plant. Similarly, P(A, / B)= OBJECTIVE TYPE QUESTIONS 1, Accouple has 2 children. The probability that both children are boys if the older one is, aboy is ayt B) + © + (D) 1 4 ®) 5 © 5 ©) 2. Acard is drawn from an ordinary pack. The probability that itis a king given that itis a face card A) i B) 4 (C) 4 D) i As Ba O FG Oy, 3. Two students A and B work independently on a problem in statistics. The probability 3 2 that A will solve tis 3 and the probability that B will solve itis 5. Then the probability that the problem will be solved is ay i py) 2 © p) 5 Oy Op Op On 4. Two dice are thrown simultaneously. The probability that the product of the numbers appearing on the top faces of the dice is a perfect square is ay B) 2 © + D4 Ws ®) 5 © 3 5 5. There are 3 red socks, 4 green socks and 3 blue socks, you choose 2 socks. The probability that they are of the same colour is at a) 2 © + p 4 Os ®) 5 © 4 OF ANSWERS LA 2A 3. Cc 4. 2B 5.D 38 Probability and Statistics & Complex Variables PREVIOUS GATE QUESTIONS Inahousing society, half of the families have a single child per family, while the remaining half have two children per family. The probability that a child picked at random, has a sibling is [GATE 2014(EQ)) A box contains 25 parts of which 10 are defective. Two parts are being drawn simultaneously in a random manner from the box. The probability of both the parts being good is [GATE2014(ME)} (A) z @) 2 ©, 3 () 3 20 125 20 9 A group consists of equal number of men and women. Of this group 20% of men and 50% of the women are unemployed, If a person is selected at random from this group, the probability of the selected person being employed is__-—_—[GATE2014(ME)] A fair (unbiased) coin was tossed four times in succession and resulted in the following outcomes : (0 Head ({i) Head (iii) Head (jv) Head. The probability of obtaining a "Tail! when the coin is tossed agian is [GATE 2014(CE)] (A) 0 B) 4 © 4 D) + “ ®) 5 ©; ©; The probability that a given positive integer lying between 1 and 100 (both inclusive) is NOT divisible by 2, 3, 5 is [GATE 2014(CS)} Let S be a sample space and two mutually exclusive events A and B such that AUB =S. If P() denotes the probability of the events, the maximum value of P(A) P(B) is [GATE2014(CS)} Suppose A and B are two independent events with probabilities P(A) #0 and P(B) #0. Let A and B be their complements. Which one of the following statements is FALSE? [GATE 2015(EQ)] (A) P(AMB)=P(A) PB) (B) P(A|B)=P(A) (C) P(AUB)=P(A) + P(B) (D) P(AMB)= P(A) P(B) Two coins R and § are tossed. The 4 joint events HpHs.TpTs,HpTs.TpHs have probabilities 0.28, 0.18, 0.30, 0.24 respectively, where H represent head and T represents tail. Which one of the following is TRUE ? [GATE2015(EE)] (A) The coin tosses are independent (B) Ris fair, Sis not (©) Sis fair, R is not (D) The coin tosses are dependent Review of Probability 39 9. 10. 1. 12. 13. 14. 15. The chance of a student passing an exam is 20%. The chance of a student passing the exam and getting above 90% marks in it is 5%. Given that a student passes the examination, The probability that the student gets above 90% marks is [GATE 2015 (ME)] at p) + © 2 pb) = Wig ®) 4 © 5 © 5 Py =4.P0) and PEXAY) =, thevalueof PCY |X) is [GATE 2015 (ME)] ay 4 py + ot p) 2 4 ®) 55 ©3 © 5 Four cards are randomly selected from a pack of 52 cards. If the first two cards are kings, what is the probability that the third card is a king ? [GATE2015(CE)] ay B) 2 © txt es ® 50 © 2%3 Candidates were asked to come to an interview with 3 pens each. Black, blue, green and red were the permitted pen colours that the candidate could bring. The probability that a candidate comes with all 3 pens having the same colouris___ {GATE 2016 (EE)] X and Y are two random independent events. It is known that P(X)=0.40 and P(XUY*)=0.7.. Which one of the following is the value of P(XUY) ? (A)0.7 (B)0.5 (C04 (D)0.2 [GATE2016(CE)] Consider the following experiment : [GATE2016(CS)} Step 1: Flip a fair coin twice Step2: If the outcomes are (TAILS, HEADS) then output Y and stop. Step 3: If the outcomes are either (HEADS, HEADS) or (HEADS, TAILS) then output N and S. Step 4: If the outcomes are (TAILS, TAILS) then go to Step I The probability that the output of the experiment is Y is (upto two decimal places) Suppose that a shop has an equal number of LED bulbs of two different types. The probability ofan LED bulb lasting more that 100 hours given that it is of Type 1 is 0.7, and given that its of Type 2 is 0.4. The probability that an LED bulb chosen uniformly at random lasts more than 100 hours is [GATE 2016(CS)} 16. 17. 18. 19. 20. 21. 22. 23. Probability and Statistics & Complex Variables The probability of getting a "head" in a single toss of a biased coin is 0.3. The coin is tossed repeatedly till a "head" is obtained. If the tosses are independent, then the probability of getting "head" for the first time in the fifth toss is [GATE 2016 (EC)] Three cards were drawn from a pack of $2 cards. The probability that they are a king, a queen and a jack is IGATE2016(ME)] ay 2& By © 2 py A) 5595 ®) 3197 Op ©) T6575 An urn contains 5 red and 7 green balls. A ball is drawn at random and its colour is noted. The ball is placed back into the urn along with another ball of the same colour. The probability of getting a red ball in the next draw is IGATE 20160) » & » o 2 » 156 ®) 156 © 156 ©) 156 Three fair cubical dice are thrown simultaneously The probability that all three dice have the same number of dots on the faces showing up is (upto third decimal place) [GATE 2017 (EQ) ‘Two coins are tossed simultaneously. The probability (upto two decimal points accuracy) of getting at least one head is [GATE2017(ME)] ‘A two-faced fair coin has its faces designated as head (H) and tail (T). This coin is tossed three times in succession to record the following outcomes : H, H, H. Ifthe coin is tossed one more time, the probability (up to one decimal place) of obtaining H again, given the previous realizations of H, H and H, would be IGATE 2017 (CE)] P and Q are considering to apply for a job. The probability that P applies for the job is 1 > The probability that P applies for the job given that Q applies forthe job is ; and 1 the probability that Q applies for the job given that P applies forthe job is 5. Then the probability that P does not apply for the job given that Q does not apply for the job is [GATE 2017(CS)} 4 5 1 u Wz ®@z ©, OD The probability that a communication system will have high fidelity is 0.81. The probability that the system will have both high fidelity and high selectivity is 0.18. The probability that a given system with high fidelity will have high selectivity is [GATE 2017(1N)] (A) 0.181 (B)0.191 (©) 0.222 (D) 0.826 Review of Probability a 24, 25. 26. 27, 28. 29. ‘Acab was involved in a hit and run accident at night. Your are given the following data about the cabs in the city and the accident. (85% of cabs in the city are green and the remaining cabs are blue. (ii) A witness identified the cab involved in the accident as blue. (iii) It is known that a witness can correctly identified the cab colour only 80% of the time. Which of the following is closest to the probability that the accident was caused by a blue cab ? [GATE 2018 (EQ) (A) 12% (B) 15% (©) 41% (D) 80% A class of twelve children has two more boys than girls. A group of three children are randomly picked from this class to accompany the teacher on a field trip. What is the probability that the group accompanying the teacher contains more girls than boys ? [GATE 2018 (EE)] ‘A) 0 p) © Dp) > @) Oia Oia OF Four red balls, four green balls and four blue balls are put in a box. Three balls are pulled out of the box at random one after another without replacement. The probability that all the three balls are red is IGATE 2018(ME)] 1 1 1 1 A) a B) 55 © 36 ©) 57 Two people P and Q decide to independently roll two identical dice, each with 6 faces numbered | to 6. The person with the lower number wins. In case of a tie, they roll the dice repeatedly until there is no tie. Define a trial as a throw of the dice by P and Q Assume that all 6 numbers on each dice are equi-probable and that all trials are independent. The probabilitiy (rounded to 3 decimal places) that one of them wins on the third trial is IGATE2018(CS)] ‘Two bags A and B have equal number of balls. Bag A has 20% red balls and 80% green balls. Bag B has 30% red balls, 60% green balls and 10% yellow balls. Contents of Bags ‘A and B are mixed thoroughly and a ball is randomly picked from the mixture. What is the chance that the ball picked is red ? [GATE 2018 (1N)] (A) 20% (B) 25% (C) 30% (D) 40% The probabilities of occurrence of events F and G are P(F) = 0.3 and P(G) = 0.4, respectively. The probability that both events occur simultaneously is PF VG) =0.2 The probability of occurrence of atleast one event P(F UG) is [GATE 2018 PD] 30. 31. 32. 33. Probability and Statistics & Complex Variables The probability that the annual maximum flood discharge will exceed 2500 m*/s,, at least once in next 5 years is found to be 0.25. The return period of this flood to be 0.25. The return period of this flood event (in years, rounded off to 1 decimal place) is [GATE 2019 (CE-FN)] An array of 25 distinct elements is to be sorted using quick sort. Assume that the pivot element is chosen uniformly at random. The probability that the pivot element gets placed in the worst possible location in the first round of partitioning (rounded off to 2 decimal places) is IGATE2019(CS)] Two numbers are chosen independently and uniformly at random from the set {1,2,3,....)13} . The probability (rounded off to 3 decimal places) that their 4 bit (unsigned) binary representations have the same most significant bit is IGATE 2019 (CS)] A box has 8 red balls and 8 green balls. Two balls are drawn randomly in succession from the box without replacement. The probability that the first ball drawn is red and the second ball drawn is green is [GATE2019 AN) 4 1 1 8 AE ®) iG Oz OG (ANSWERS 1, 0.667 200A 3. 0.65 4. B 5. 0.26 6. 0.25 2c 8 D % B 10. C 1B 12, 0.01818 13. A 14, 033 15. 0.55 16. 0.07 17, A 18, A 19. 0.028 20. 0.75 21.0.5 2A 23. C 24. C 25. B 26.B 27. 0.023 28. B 29. 0.5 30. 17.9 31. 0.08 32. 0.461 33. A CHAPTER - 2 2.1 INTRODUCTION Previously we considered the concept of random experiment, events, sample space and sample points. The events described on a random experiment may be numerical or non-numerical (descriptive). For example, the outcomes that we obtain, when we throw a die are numerical. (We get the outcomes as 1, 2, 3, 4, 5 or 6). Hence the sample space is numerical. But, the ‘outcomes we obtain, when we toss a coin are non-numerical. We get the outcomes as head or tail. Here the sample - space is non-numerical or descriptive. It is inconvenient to deal with these descriptive outcomes mathematically. Hence, for ease of manipulation, we may assign a real number to each of the outcomes using a fixed rule ‘or mapping. For example, when we toss a coin we get two outcomes, namely head or tail; we can assign numerical values, say ‘1’ to head and ‘0° to tail. ‘This interpretation is easy and attractive from mathematical point of view and also practically meaningfil This rule or mapping from the original sample space (numerical or non-numerical) to a numerical (real) sample space, subjected to certain constraints is called a random variable. Thus Random variable is a real valued function which maps the numerical or non-numerical sample space (domain) of the random experiment to real values (codomain or range). 22 RANDOMVARIABLE We know that a variable is a quality which changes or varies - the change may ‘occur due to time factor or any factor. For example, the height of a person vary with age, but the age ofa person changes with time. Every variable has a range in which it can take any value. Examples are: marks scored by a student in an examination, number of children in a family, height and weight of a person, etyc. The variable could be continuous or discrete. A continuous variable takes all possible values in its range. For example, the height of a person can take any value in a certain range, but for the sake of convenience, it is measured only up to the accuracy of inches or cms. Similarly, the weight of a person can be any value, but it is usually expressed in kgs. A discrete variable takes only certain values in a range. For example, number of children in a family, number on a dice, ete can take only integral values. A variable is called a random variable when there is a chance factor associated with the various values, which it can take in its range. For example, the life of a person is a random variable and there are probabilities associated with various age values. 43 44 Probability and Statistics & Complex Variables Def. : A real variable X whose value is determined by the outcome of a random experiment is called a random variable. Arandom variable X can also be regarded as a real - value function defined on the sample space S of a random experiment such that for each point x of the sample space, f(x) is the probability of occurrence of the event represented by x. [JNTU (H) Dec. 2014] Illustration: The sample space corresponding to tossing of two coins. When two coins are tossed, its outcomes or sample points can be (Head, Head), (Head, Tail), (Tail, Head), (Tail, Tail) i.e., S= {HH, HT, TH, TT}. After the performance of the experiment, we count the number of tails and denote it by X. The first outcome HH has 0 Tail, so X = 0. Similarly, X = 1, denote the outcomes HT or TH and X = 2, represents the outcome TT. Thus X takes the values 0,1,2 i.e., X = 0,1,2. A random variable is also called a stochastic variable or simply a variable or a chance variable. The name random variable is given to the variable X because it is defined on a sample space, the outcomes of which are uncertain and hence depend on chance. The random variable assigns a real value X(w) such that 1, Theset {w| X(w) R by X(s) = “number of heads”. Then X(s,) = 2, X(s,) = 1, X(s,) = 1 and X(s,) = 0 Range of X = {X(s) : seS} = {X(6)), M6), XC), M6} = {0, 1,2} Ex. 2. Consider the random experiment of throwing a pair of dice and noting the sum. The sample space consists of points S = {(1, 1), (1, 2), «+ (1, 6), (2 1), (2,2), 2, 6), sony (6 1s (6, 2)5 ees (6, 6)} Random Variables 45 If.X is the “sum of numbers appearing on the faces of dice”, X is a random variable. Here X:§—>R is defined through X((i,j)) = i + for (i,j) €S. If in a random experiment, the event corresponding to a number a occurs, then the corresponding random variable X is said to assume the value a and the probability of the event is denoted by P(X= a). Similarly, the probability of the event X assuming any value between a and b is denotd by P(a < X c is denoted by P(X > c). That is, P(X > c) is simply the probability of the set of ‘outcomes s for which X(s) > ¢ or P(X > c) = P{s |X(s) > c}- For the example (1) given above, we have 3 P(X < 1) = P(HH, HT, TH} = 4 If X and Y are random variables on the sample space S, then X + Y, X + K, KX and KY (K is a constant) are also random variabels on $ and are defined by (X+ ¥) (s) = X(s) + M(s), (K+ K) (5) = X(6) + K, KX (8) = K. X (9), ete. Note : 1.P(X< c) + P(X> c) = P(-2 0) =1-P(X 0 for all values of §; 0< pj <1 (ii) Zp) = 1, 7 = 1, 2, 3, oy then the function p(x) is called the probability mass function of the random variable X and the set {p (x,)}, /= 1, 2, ... is called the discrete probability distribution of the discrete random variable X. The probability distribution of the random variable X is given by means of the following table: x [alels P(X) | pi | Pe | Ps Further P(Xx)= 1-PX Sx) Ex. 1. In tossing a coin two times, $= {7T, HT, TH, HH} P(X = 0) = Probability of getting two tails (no heads) 11 1 = PUT T= 5-5 > 5 P(X= 1) = Probability of getting one head 1,101 = PULTE) = 4 tga P(X = 2) = Probability of getting two heads Ble = PC{H, H}) = til Thus the total probability | is distributed into three parts 3, 5, 4 according to whether X=0 or 1 or 2. This probability distribution is given in the following table : x o}1}2 Vy] MX=%)19|5/9 Random Variables 49 Ex. 2, Let X be the random variable which represents the sum of numbers of points on throwing two unbiased dice. If the point shown in each dice is equal to one, the minimum sum of numbers is equal to two. If both dice show six, then the sum of numbers at the maximum is 12. Thus if two unbiased dice are thrown, then the sum X of the two numbers which turn up must be an integer between 2 and 12. For X= 2, there is only one favourable point (1, 1) and hence P(X = 2)= 1/36, since there are 36 sample points in all. For X= 3, there are two favourable sample points (1, 2) and (2, 1) and hence P(X = 3) = 2/36. Similarly for X = 4, there are three favourable sample points (1, 3,), (2, 2), (3, 1) and hence P(X = 4) = 3/36 and so on. Now the probability distribution in this case is given by the following table: x=x, |2/3|4 {5 |6|7]8 | 9/10 | mu {12 P(X =x) | 1/36 | 2/36 | 3/36 | 4/36 | 5/36 | 6/36 | 5/36] 4/36| 3/36 | 2/36 |1/36 Clearly Bp(x) = py + Py tm * Py = From the above table, we have 155 36° 36-12 4 3 2 4 7 eee ee 36° 36° 36 36 «18 In the above example, X takes only a finite number of values as such it is a discrete random variable. 5 and P(T> p(x) i.e, f(x) denoted by F(x) is called the Cumulative = Distribution Function or simply Distribution Function. It is similar to cumulative frequency i.e., the cumulative frequency upto a given point or value. It gives the probability that the random variable takes any value from 0 (lowest value that the variable can take) to agiven value. For example, 2 F)= DY /(x) = £0) + (+ £2) = p(O) + pl) + p(2) att -4at2tG Suppose if X takes only a finite number of values x,,.x,, ..., then the distribution function is given by =1 (in Ex. 1 above) Ow (Kx); = KD. % Ps = K EC), where K is a constant. fa | (ii) If.X is a random variable and a and 6 are constants, then E(aX + b)=aE(X) +b Proof: Proceed as in (i) (iii) If X and Y are any two random variables, then E(X + Y) = E(X) + E(¥) provided E(X) and E(Y) exist. [JNTU 2005 S, 2005, 2007S, (H) Nov. 2009 (Set No.1)] Proof : Let X assume the values x,,%,,..., and Y assume the values y,, Yay Dy Then by definition, E(X) = > P.% and E(Y) = bn yy Y= y)= poy) [This is called the joint probability function of X and Y] The sum (X + ¥) is also a random variable which can take m * n values (x, + y,), 52, ey mand j = 1, 2, By definition, Let p, = PX EX + = = Det D yyy = wey + 20) iat Note: 1. E(X + ¥+ Z) = E(X+ (¥ + Z)) = E(X) + E(¥ + Z) = EX) + B(Y) + E(Z) Similarly proceeding, we have in general by the method of induction, if XyXqy wnyXq are nm random variables then E(X, + X3 tot Xq) = B(Xq) + B(X3) ++ (XK) i.e., The mathemtical expectation of the sum of random variables is equal to the sum of their expectations, provided all the expectations exist. 2, E(aX + bY) = a E(X) + b E(Y), where a and b are constants 3. E(X- X)=0 (iv) If X and ¥ are two independent random variables, then BUY) = EQ) EW) Random Variables 53 Proof: Let X assume the values x,, x,, ..., x, with respective probabilities Py Py + P, and ¥ assume the values y,, Y35 +), With probabilities 7, pb...» Ply Then by definition, F(X) = DP and EC) = P59 a a Let py= PU=x,¥=y) PUX= x). P (Y= y)) (since X and Yare independent) = PPh The product X ¥ is a random variable which can assume m x n values x,y, cop mand j = 1, 2, 15m. ©. By definition, xy) DL =L Vay ar af A (& » | (é wn} = FU) EW) Note : (XYZ) = E(X.YZ) = E(X) . E(¥Z) = E(X) E(Y) E(Z) Most of the concepts discussed with the frequency distributions apply equally well to distribution functions. ; 1 1 Important Observation : (3) and cp are not same. (2) Mean : The mean value p1 of the discrete distribution function is given by = Px, we Zp tee = EO) Note: If E(X)=, then E(¥-p)=0. (3) Variance : Variance characterizes the variability in the distributions since two distributions with same mean can still have different dispersion of data about their means. Variance of the probability distribution of a random variable X is the mathematical expectation of [X - E(X)]°. Then Var (X) = E[X -E(X)P ice., Var (X) =¥{fs-200P xpd} al Another Form of Variance : If X is a random variable, then the mathematical expectation of (X —)° is defined to be the variance of the random variable X. Then Note (4) Probability and Statistics & Complex Variables Var (X) = BO = w=) a -H)? t= Mw. [en=Zxp.Ep,=0 2 8 =E(X?)-p? ie.| Var (X) = EQ?) - (ECO? The variance of a random variable X is also denoted by V(X). Standard Deviation : It is the positive square root of the variance. dest = VEX?) =? = JE[X- ECO] © S.D= Some Important Results on Variance : 1. Variance of constant is zero i.e., V(K)=0 2. If K is a constant, then V(KX) = K?V(X) 3. If X is a random variable and K is a constant, then V(X +K)=V(X) 4. IfX is a discrete random variable, then V(aX + 6) = a? V(X), where V(X) is variance of X and a, b are constants. Proof. Let Y= aX+b ~@ Then B(Y) = E(aX + b) = aE(X) +b » Q) (1) = Q) gives Y- E(Y) = a[X - EX] ‘Squaring and taking expectation of both sides, we get EL(Y — BYP) = @EL(X — EUD}?] Le, VO) = a? VON) or V(aX+ b) = a? VOX) Cor. (i) If b=0, then V(aX) = a® V(X) (i) Ifa =0, then V(b) =0 (iii) Ifa = 1, then WX + b) = VX) 5. IfX and Y are two independent random variables, then V(X +¥)=V(X)+V(¥). [REMIT L (1/2), find the minimum value of K and, (iv) Determine the distribution function of X (v) Mean (vi) Variance. [JNTU 04S, 05S, 08S, (A) Nov. 11, (H) Dec. 11, (K) May 10S, Nov. 2012, Mar. 2014 (Set No. 2)] Solutio 7 () Since 2) PC) = 1, we have = K+2K+2K+3K+K?+2K?+7K?+K=1 ie, 10K? + 9K-1=0 ie. (10K -1) (K+ 1)=0 ake pro (since p(x) > 0, so K ~ 1) (ii) PIX < 6) = PX = 0) + P(X= 1) +... + P= 5) = O+K+2K + 2K +3K + K= 8K + K= 0.8 + 0.01 = 0.81 [e K=0.1] P(X> 6) = 1- P(X <6) =1-0.81 = 0.19 P< X <5) = PX = 1) + P(X = 2) + PX = 3) + PX = 4) P(OS X <4)= P(X =0)+ P(X =1)+ P(X =2)+ P(X =3)+ P(X =4) =04+K 42K +2K +3K =8K =8(0.1)=08 Note: P(X <5)= P(X =0)+ P(X =1)+....+ P(X =5) .81 (Refer (i)] P(X >5)=1- P(X $5)=1-081=0.19 P< X <6)= P(X =1)+ P(X =2)+...+ P(X =5)=0.81 (iii) ‘The required minimum value of K is obtained as below. Le a P(X <2)= [P(X = 0) + PUX= 1)] + P= 2) 8 PIX <1)= P(X =0)+ PX=1)= 04+ K= 0.1 Random Variables 87 10 P(X <3) = [P(X= 0) + P(X= 1) + P(X = 2)] + P(X = 3) = 0.3 + 0.2=0.5 PIs 4)= PU S3) + P(X=4) = ose = os>0s= 4 1 +, The minimum value of K for which P(X < K)> 5 is K= 4 (i) The distribution function of X is given by the following table x FQ) = PS) | 0 1 K=1/10 2 3K = 3/10 3 5K =5/10 4 8K= 8/10 5 8K + K?= 81/100 6 8K + 3K? = 83/100 1 9K + 10K?= 1 1 (0) Mean, w= 2% = 0(0) + 1(K) + 2(2K) +3(2K) + ACK) + 5(K2) + 6(2K2) + 7(7K2 + K) 66 30 1 = 66K? + 30K = FE + P= 0.66 +3 = 3.66 oxo) 1 (vi) Variance= pix? — i = K+ 8K + 18K + 48K + 25K? + 72K? + 343K? + 49K ~ (3.66)* = 440K? + 124K - 3.66) = 4. 74 _ G66 = ~ 8.66)" = F099 + Fo ~ G86) = 44+ 12.4 - 13.3956 = 3.4044 |A random variable X has the following probability distribution. x [1 [2 [3 [4 [5 [oe [7/8 poy | K | 2K | 3K | aK | SK | &K | K | 8K Find the value of @ K (i) P(X <2) (iti), PAE X <5) —_[INTU2008(Set No. 1)] Solution: (i) Since D) P(%) = 1, we have ial K+2K+3K+4K +5K+6K+7K+8K=1 = 36K=1. 58 Probability and Statistics & Complex Variables (i) P (XS 2) =P(K=1)+ PX =2) 3 1 1 HRI = K= 3-7 [E35] (ii) P22 < X <5) =P(X =2) + P(X =3) + P(X =4) + P(X =5) =2K+3K+4K+5K= 14K 47 36 «18 Arandom variable X has the following probability distribution. Valuesofx[0]1]2[3]4[5]6]7]8 P@) [a |3a|Sa| 7a [9a [ta | 13a [15a [170 (i) Determine the value of a. (i) Find p(x <3), p(x23) and p(0 p(x) where x is any integer a Random Variables value of k so that P(X <2)>0.3? (ii) (ii) x F(X) =P(X sx) “i 1 2 3 16a = 76 4 2502 5 36a = 36 6 490-2 1 8 The probability density function of a variate X is oft 2 3 [,4]s [eo k | 3K 3k 7k | 9K | 1k | 13% () Find k (if) Find P(X<4), P(X > 5), PG0.3= 9k >0.3 or bor :. The minimum value of k is 35 Probability and Statistics & Complex Variables For the discrete probability distribution xX jo} 1} 2,3) 4 5 6 P(X) [0 | 2K | 2K [3K | K? | 2k? | 7k? +K. ind (i) k (i) Mean (iii) Variance [INTU (Ht) 2009 (Set No. 1)] é Solution: (i) Since the total probability is unity, we have 2, P(*)= m ie, 0+2K +2K +3K +K? +2K? +(7K?+K) 4.25 10K? +8K— 8+ V64+40 _-84VI04 _ 84226 _ -44 26 20 20 20 10 Since, p(x) 20, W4+V26 = 0.1099 10 « (ii) Mean, 1 = >) 7.x; B = (00) + (2K) #(2)(2K) + (3) BK) + (4K?) + (9)(2K*) + X7K? +K) =2K+4K 49K +4K? 410K? +42K? 46K = 56K? +21K = K(56K +21) = (0.1099)[56(0.1099) + 21] = 2.9842 ‘ (iii) Variance = ¥° pjx? =? = 0+2K(I)? +2K(2? +3K@)" + K?(4)? +2K7(5)? + (7K? +K)(6)? ~ 2.9842)" = 2K +8K +27K +16K? +50K? + 252K? +36K -8.9054 =318K? +73K —8,9054 = K(318K +73)—8.9054 = (0.1099)[318(0.1099) +73]-8.9054 = 2.9581 A random variables X has the following probability function x, |-3] 2] 1] 0] 1 | 2 3 px) | K | or] K [02] 2¢ |o4 | 2% Find()K (ii) Mean (iif) Variance [NTU (Ht) 2009 (Set No. 2)] Random Variables 64 Solution : (i) We know that, if X is a random variable, then Y p(x,)=1 ie, K+0.14+K+0.2+2K+0.442K =1 ie, 6K+07=1 ie, 6K=03 or K ‘ (i) Mean, w= > 7.x 3 = (-3)(K) + (-2)(0.1) + (—1(K) + 0(0.2) + 1(2K) + 2(0.4) + 3(2K) =-3K-0.2-K+0+2K+0.8+6K =4K+06=4(%)-06 =08 “ (iii) Variance = ¥) pix? - 0° = K(-3)? + (0.12)? +K(-1)? + 0.2(0)? +2K (1)? + (0.4)2)° +2KG)" - 0.8) =9K +0.4+K +0+2K +1.6+18K -0.64 =30K +2-0.64 =30(%!) 1 36 =1.5+1.36=2.86 From a lot of 10 items containing 3 defectives, a sample of 4 items is drawn at random. Let the random variable X denote the number of defective items in the sample. Find the probability distribution of X when the sample is drawn without replacement. [JNTU (K) May 2013 (Set No. 4)] Solution : Obviously X can take the values 0, 1, 2 or 3. ven total number of items = 10 No. of good items = 7 No. of defective items = 3 1 CT ate PUK = 0) = Pine detective) = yr = Gj * Toy = P(X = 1) = P(one defective and 3 good items) 5.x7Cy _ 3x7! 4t6! We, 3a! * Tor P(2 defective and 2 good items) 3Gx'G _ 3 "Cy 10 P(X=2) 62 Probabilly and Statistics & Complex Variables P(X= 3) = P(3 defective and 1 good item) Gx"G 7 at 1 ™C, "C, ~ 8x9x10 30 £. The probability distribution of random variable X is as follows x]ofif2 ]3 1 1 3 1 6 | 2 | 10 | 30 Let X denote the minimum of the two numbers that appear when a pair of fair dice is thrown once. Determine the (@ Discrete probability distribution (ii) Expectation (ii) Variance JINTU 2006, (K) Nov. 2009, 20108 (Set No. 1)] Solution: When two dice are thrown, total number of outcomes is 6 x 6 = 36 PX) (1,1),(1,2),(1,3),(1,4)5(1,5),(,6) (2,1),(2,2),(2,3),(2,4), (2,5),(2,6) (3,1),(3,2),.3),(3,4),(3,5), (3,6) (4,1),(4,2),(4,3), (4,4), (4,5).(4,6) (5,D.(5,2),5,3),6,4),(5,5),(5,6) (6.1), (6,2),(6.3), (6.4), (6,5), (6,6) In this case, sample space S= If the random variable X assigns the minimum of its number in S, then the sample space S= RR eee wuERene aAuwaens The minimum number could be 1, 2, 3, 4, 5, 6. For minimum 1, favourable cases are (1, 1), (1, 2), (2, 1), (1, 3), G, 1), (1, 4), 4, Ds (5). (5, 1), A, 6), ©, 1). For minimum 2, favourable cases are (2, 2), (2, 3), (3, 2). (2, 4)s (4, 2) (2, 5) 5 (5, 2), (2, 6), (6, 2). 9 So P(X = 2) = Random Variables 63 (ii) (iii) Note Similarly, P(X = 3)= P((3, 3), (3, 4), (4, 3), (3, 5). (5, 3)s (3, 6), (6, 3) = 36 P(X = 4) = P(A, 4), (4, 5), (5, 4), (4, 6), (6, 4)) = 2 36 PX = 5) = P((S, 5), (5, 6), (6, 5)) = 36 P(X = 6) = P((6, 6)) = 3 2. The probability distribution is x 1 {2 |3 |4 [5 }6 PU) MY] 7) 5S] 3) : 36 | 36 | 36 | 36 | 36 | 36 Expectation = ie, BO) = or op = z (11+ 18 +21 + 20+ 15 +6)= 2 =25978 Variance = © p,x?- u 9 5 3 91 = His Sar Zor Ser dase bae-(2Y a H (1 #364634 80+ 15+ 3 -(2J ie, o? =8. 3611 ~ 6.3898 = 1.9713 + Standard deviation, o= VI.9713 = 1.404 Calculate expectation and variance of X, ifthe probability distribution of the random variable X is given by @ x ito |i [2 [3 f | 03] 01] 02] 03] 02 [INTU 2006 (Set No.2)] Solui Expectation = Mean = 5 fx, i.e.,E(X) = ~ 1) (0.3) + 0(0.1) + 1(0.1) + 2(0.3) + 3(0.2) or #=-0.3+0.1 +06 +0.6=1 64 Probability and Statistics & Complex Variables (i) Variance = & f,x?- p= 2x? f- [EOP © 1)? (0.3) + 0(0.1) + 1(0.1) + 27(0.3) + 3°(0.2) — 1 0.3+0.1+12+18-1=24 Calculate the mean for the following distribution. X=x 03 {02/01 | o/1 |2 |3 PX=x): | 0.05] 0.10] 0.30] 0 | 0.30] 0.15] 0.1 [JNTU 2008 (Set No. 4)] Solution : The mean value of the probability distribution of a variate X is commonly known as its expectation and is denoted by E(X). It is given by EX) = D xf) = 0.3(0.05) + 0.2(0.10) + 0.1(0.30) + 0(0) + 1(0.30) + 2(0.15) + 3(0.1) = 0.015 + 0.02 + 0.03 + 0 + 0.30 + 0.30 + 0.3 = 0.965 [EUMIMEEIY random variable X is defined as the sum of the numbers on the faces when two dice are thrown. Find the mean of X. [INTU (A) 2009 (Set No. 4)] Solution: Let x be the sum of the numbers on the faces when two dice are thrown. x is a discrete random variable whose probability distribution is given by 273 [4 [5s [6 ][7f.[u [2 x; (x;) | 1/36 | 2/36 | 3/36 | 4/36 | 5/36 | 6/36 .... |2/36 [1/36 Mean of x = E(X) =) p,x; = lse)el ese) is) “ae)U58) “a se)ose) +olGe) Ge) °Lse) = 7gl2+6+124 204304 42+ 40-+36430422-+12) = 2825 36 Find the mean and variance of the uniform probability distribution given 1 by f(a) == forx= 1, 2,3, 4M. [INTU 2001, (H1) Nov. 2009, Nov. 2010, Dec. 2011 (Set No. 3)] Random Variables 65 Solution : The probability distribution is x 1 2 3 1” n vfaja 1 fo n n n 7 n a 14t @ Mean = x, f(x;) = Lt +n. ‘1 or6t) = y= Laszsse.te (ii) Variance = xy xP (x)- i = 1 nme Qn+1) lays yy n 6 _ (nt Qntl) (n+)? (ae (2as1_nel) - 6 “4 203 02 _ andl _ 4 (n-t) _ = Ap nt 2 3n 3) = = [EXDINIEAEED 4 sample of 4 items is selected at random from a box containing 12 items of which 5 are defective. Find the expected number E of defective items. [JNTU (H1) Nov. 2009 (Set No. 4), (A) Nov. 2010 (Set No. 1)] n : Let X denote the number of defective items among 4 items drawn from 12 Obviously X can take the values 0, 1, 2, 3, or 4. No. of good items = 7 No. of defective items = 5 een "ec, 495° 99 P(X=0)= P(no defective) = P(X=1)= Plone defective and 3 good items) 1Cyx °C, _ 1x6x5x5 115 _ 35 PCy 6 495 99 66 Probability and Statistics & Complex Variables P(X=2) PQ2 defective and 2 good items) 210 _ 42 195 99 P(X = 3) 1 3 S zB 8 a 3 P(X=4)= P(all are defective) =e Sd “Te, ~ 495 99 Discrete probability distribi x, o ji f2]3 [4 7 | 35| 42] 14] 1 P(X =x) =f (x) 99 | 99| 99] 99] 99 Expected number of defective items = E(X) = E x,f (x) Zi Ba. BisBigt 18 99 99 99 99 99 (99 Show that the variance of a random variable X is given by o? = E(X) ~ E007 We know that oF = ELK w'] = BU? - 2 X w+ w) = BUY) - 2p BCD + EQ) = BG?) = pg + p= EO?) ~ 42 = BOP) ~ [ECP Given that f(x) =K/2x, is a probability distribution for a random variable X that can take on the values x=0,1,2,3 and 4. (i) FindK (ii) Mean and variance of X. [JNTU (A) Sup., 2010) Solution : Given f(x) = K/2x , where x=0,1,2,3,4. If x=0,/(x) is not defined Hence the problem is not correct. ‘We shall delete x =0 and workout the problem. () If f(x) isa probability distribution function, then Xe0>K20 1. £@)20> f@)=s2 Drw=193y * .Yr@=12y * 2% ya 2x Random Variables The probability distribution is given in the following table : x {1 f(x) cel | 2/3 es 4|6 rs (ii) | Mean = F(X) Suse A Now E(x?) Se SQ) = atx Kg K 9K 160K 2 4 6 8 Kak ox sk 2 2 ». Variance = E(X?)-[E(X)P =5K - (2K) = 5K -4K? =K(5-4K) =(3-25) [ex -2] 25 25 24 2) 696 = 2) 08 ii 3 (5 as 11156 variate X has the following probability distribution x | -3[6] 9 r}aja Po) |g | a) 3 Find (i) E(X) (ii) B(A") (ivi) AU(2X + 1) Probability and Statistics & Complex Variables Solution : . < 1 1 1 ul @® BY = Dax = ca(retrat -3t3e3e7 3 1 1 1 (i) Bee) = Dp = Cy (3) +O. 5+ OG ial 3 81_ 93 6 32 (iii) E[QX + 1] = EX? + 4X + 1) = 40%) + 4) + EC) 93 u 418 9 tag thay = 209 A player wins if he gets 5 on a single throw of a die, he loses if he gets 2 or 4. If he wins, he gets Rs. 50, if he loses he gets Rs. 10, otherwise he has to pay Rs, 15. Find the value of the game to the player. Is it favourable ? Solution : Range of X= {- 15, 10, 50} Since there are six numbers in a die, out of these 5 is only one number, the probability of getting Rs. 50 is 9, 36 = s+Se 2 1 PUX= 50) = & Similarly, the probability of getting two numbers (2 or 4) to win Rs. 10 is a= 10)= 2-4 Cine The probability of getting three numbers (1 or 3 or 6) to loose Rs. 15 is PX=-15)= E=5 Discrete probability distribution is, X=x 15] 10 | 50 1 1 1 raH=r | 5 13 | ¢ Expected value of the game = E(X)=E.x,p (x)= (15.4410 4 +50 ie Rs. 2 Game is favourable to the player since E > 0. A player tosses 3 fair coins. He wins Rs. 500 if 3 heads appear, Rs. 300 if 2 heads appear, Rs. 100 if 1 head occurs. On the other hand, he loses Rs. 1500 if 3 tails occur. Find the expected gain of the player. Solution: Let X denote the gain. Then the range of X is {~ 1500, 100, 300, 500} The sample space S =n(s) = 23 = 8 = {HHH, HHT, HTH, HTT, THH, THT, TTH, TIT} Random Variables The probability of all 3 heads (getting Rs. 500) = P(X = 3) = ele The probability of getting 2 heads (getting Rs. 300) = P(X = 2) The probability of getting one head (getting Rs. 100)= P(X = 1) = 1 The probability of getting 3 tails (lossing Rs. 1500) = PX = 0) = rete probability distribution is, X=x, — 1500 | 100 | 300 | 500 1 {3 [3 [a P(X = x)= p(x) 8 is [8 [8 Expected value of X = £(X)= 2 x, p(x) 4 + 100.2 + 300.2 + 500.2 8 8 8 g 1 = © 1500 + 300 + 900 + 500) = Rs. 25. [EEEIIIETED 4 fair coin is tossed until a head or five tails occurs. Find the expected number E of tosses of the coin. [NTU (HD) Nov. 2009 (Set No.3)] Solution: If head occurs first time there will be only one toss. On the other hand, if first one is tail, second occurs. If head occurs there will be only two tosses. Suppose second one is also tail third occurs. If head occurs there will be three tosses and so on. (1) = p(H) = ©1500). » P(2)= p(TH) PQ)= pOTTH) =, pa)= peTTTH) = 5 (5) = p(TTTH) + p(TTTTT) = + 3232-16 The probability distribution function of X is x 1 2 3. [4 5 Ba a Af yf PO) 2 4 s_| is | ie Hence E(X) =} px, wlxtgaxtaaeteaxtsed aby 2y Dg eg 2a 848464445 _31 = 22 =1.9375 16 16 70 Probability and Statistics & Complex Variables PREIMIGETEY 4 player tosses two fair coins. He wins * 100/-ifhead appears, * 200/~ if two heads appear. On the other hand he loses * 500/- if no head appears. Determine the expected value E of the game and is the game favourable to the player? [JNTU (K) Nov. 2009 (Set No. 3)] Solution : Let X denote the number of heads occuring in tosses of two fair coins. The sample space S is S= (H,T} x {H,T} = {HH, HT, TH, TT} Probability of all 2 leads= P(X = 2) Probability of all 2 tails = P(X =0) = IN ale Probability of one head = P(X = Discrete probability distribution is X=x,] 0 I 2 | my | | 2 1 i) |G al 2 1 1 Expected value of the game = 100% + 200x 5 - 500% _ 200+ 200-500 _ -100 ga Game is not favourable to the player since E <0 [ESDIIIEERD 4 discrete random variable X has the following distribution function 0, forx <1 1/3, for lx <4 F(x) =} 1/2, for 42x <6 5/6, for 6 10 Find (i) P(2 (i) The probability distribution of X is x [2]3]4]5]6 PG)) 1) 2/3) 241 9|ol9lolo 12g 3.6 2) i) Mean = E(x) = = 2x 43x 44x 45x 46x (i) Mean = E(x) == px; xg t3xG tdxg +x 5 + 6x5 u (2464124104 6)=136)=4 (iii) B(x?) =¥ pix? abd 2943x164 2x25+4%36 go 9 9 9 1 1 = hid 1844850436) = 48-1733 Hence variance = E(x?)—[E(x)P =17.33-(4)? =17.33-16 =1.33 REVIEW Q| IONS) 1. Define a Random Variable. [JNTU (H) Dec. 2014] 2, Write the definitions of (i) Random variable (ii) Discrete Random Variable (ii) Continuous Random Variable and (iv) Probability Distribution function [JNTU (K) Nov, 2011 (Set No. 2)} 3. Define random variable, discrete probability distribution, continuous probability distribution and cumulative distribution. Give an example of each. [JNTU 2007, 20085 (Set No.4), (K) Nov. 2011 (Set No. 1)] 4, List the properties of probability distribution function. [JNTU (K) Dee. 2013 (Set No. 3)] 5. What is the Mathematical Expectation ? 6. Write the properties of Mathematical Expectation. 7. (a) Define (i) Probability density function (ii) Probability mass function (iii) Discrete random variable (iv) Continuous random variable [JNTU (K) Dee. 2013 (Set No. 3)] (b) Explain with suitable examples, discrete and continuous random variables. [JNTU (H) Nov. 2015] Random Variables 75 EXERCISE 2(A) 1.(a) The probability density function of a variate X is as follows =x]0 [1 [2 [3 [4 [5 |6 P(x) |K 3K_| 5K | 7K | 9K | 11K} 13K (® Find P< 4), PUC> 5), PB 0.3. JSNTU (K) Nov. 2009, Mar. 2014 (Set No. 4)] (6) Arandom variable X has the following probability function : xX fofif3[4 [so | Px) | o| K [2K | 2x [3x | x? [7x7 +K| (i) Find the value of K (ii) Evaluate P(X <6),P(X 26) (iii) Evaluate P(02) (v) P(x<2) (vi) P(-10 _. . LO)" 10, forx 05) = J fede = J Se)dr+f seyae os 1 cae) 3 (2 = 3 (2-0.4583) - 3(3 0.4583) 0.3125 81 82 Probability and Statistics & Complex Variables The probability density f(x) of a continuous random variable is given by S@) e", -o0 < x < 00, Show that c = 1/2 and find the mean and variance of the distribution. Also find the probability that the variate lies between 0 and 4 [JNTU Jan. 2007, (A) Nov. 2010 (Set No. 2), (K) May 2013 (Set No. 4)] Solution: Given f(x) = ceM,—a -2¢(0-1) > 2e=1 we= Hence f(x)=ce™= 5 eH = J fade (Mean of the distribution, E(x)= jt ; J xe “dx =0, since integrand is odd. (ii) Variance of the distribution, ° ® 1 = fw saya = f (05 6a -1 feet = 2.5 [xe dx, since integrand is even 0 Nie Random Variables 83 = jee dx = (: (0-2) =2 (iii) The probability between 0 and 4 = P(0< X< 4) i¢ lft gy fetlde => fete Eoin x< 4 |al=x] a ° Lyon 1a 2 Meg Me) 1 = F(1 ~ €°4) = 0.4908 (nearly) Probability density function of a random variable X is 1 =sinx, forOsx fjesinew 5 (ii) Mode is the value of x for which f(x) is maximum Now f'(x) = ; cos X For f(x) to be maximum, f"(x) = 0 x ie, cos x=0 84 Probability and Statistics & Complex Variables (xy =— + sine. ate = %, p* 12g SMa) == 7 sine. At =F, IM) == 9 Nia Hence f(x) is maximum atx = x + Mode of the distribution is given by x = > (iii) If M is the median of the distribution, then M 2 1 J fedae = f fede == a M 2 wy tl 1 fon J ysinx de = J ysine ae oF 0 iu => l-cosM=1 = cosM=0 .M Median of the distribution = Thus Mean = Mode = Median = RIA als = =2y f feydc= f sine dx o 0 (i) PO ex0 £0)* Vo, otherwise Determine (i) k (ii) Mean (iii) Variance [INTU (A) Dec. 2009, Nov. 2010, Dec. 2011, (H) May 2011, Nov. 2012, (K) May 2013 (Set No. 1)] Solution : (i) Since the total probability is unity, we have Random Variables 85 0 2 * ie, [Odr+fire™dr=1 ie, k fre dr=1 a <» 0 MSE tesk [0--(0-4] = 1ork= 4 Now f(x) becomes _ [xe for x 20,2 >0 LE)= Yo, otherwise (ii) Mean of the distribution, w= [x f(x) dr ° 2 e jew = | Odr+ [xatre Made = 92 [xr eo Mdr 0 “2 0 as ois ois = 92| "|< ]-2x| S-]+2] S|] using Bemoulli’s Rule ay ze ae || + using 2 2|(0-0+0)-(0-0 oS 2 +0)-( | N (iii) Variance of the distribution, o? =f x°/(x) dx— p> 2 Pe te,o= ff soa-(3) =12 fe ae (Apply Bernoulli's Rule) 0 ° oe = gle{S -» (SF 2 [(0-0+0-0)-(0-040-)} = 86 Probability and Statistics & Complex Variables continuous random variable Xis defined by zl 2 eta if -3 0 for every x in [-3, 3] and @ 3 1 1 3 * J feyar = f separs f sopacs | pepar+] poyaet f feo de ey -o 3 a i 3 304 1 3 1 2. 1 2 1 2. = J Ode+ fise+ de + Sis6-2 det fig) aloes 1 27 1 2 1 [@-» 2 +752 16 deed S | 465) Hence the function f (x) satisfies the requirements for a density function. feel 3037 Mean of f (x) = i x f(x)de 17 ut 14 5 ig Lr0+ 7 ee+ig 6-23) e+ ig [20-29 a Random Variables “1 3 = frre +60 de40+ 1 [x0-6e+22) de i [-» The integrand of the second integral is odd function] 17 243 1} 243 S ig) Ort" tex dae ig] x6 +0 )de 1 (92, 6? 16 \ 2 3 =0 PRETTIIERY 's the function defined by fox<2 1 S@)= yg Ort32sx54 0, x>4 a probability density function? Find the probability that a variate having (x) as density function will fall in theinterval 2 0 and 2 e 4 e J f(a) de = [odes | g(ex+)ere [0d _1t 1 oxss] oF Jor+dar Sag [exe 1 = 5y (121-49) =1 Hence f (x) is a probability density function. (ii) The probability that the density will fall in the interval 2 5) {SNTU 2003] Nie (ii) Find a number & such that P(X < k) = Ww ww Solution : (i) e(x<3)= f feyae = f 02x -21:7 +108) ae 0 a vow r(x>4)-1-P( xs (ii) Given P< 8) = ; r k ie, free =i ie, Jaae a? +100 de=3 41d 45x2y¢ <2 ie, Gr - Te +52 = 5 1 .e.3K— 7h + 5B => or 6K = 14K + 10-1 =0 k= 0.452 ‘Accontinuous random variable X has the distribution function 0, if xsi F(x) =} k(x-1)', if Lexs3 Lif x>3 Determine (i) f(x) (if) k (iii) Mean [JNTU 2004S, 2007S, (A) Apr. 2012 (Set No. 2)] Solution : (i) We know that f(x) = 4 Fol ix Random Variables 89 O,ifx st SL) = 44k(x-1P,ifl3 (ii) _ Since total probability is unity, we have 3 3 J/eodr=1 ie, ak [e—Ddr=1 1 1 7? ¥ =l1 k(16-0)=1 or J Ht fen kU6-O)=1 of k= 76 0, ifxst 1 Hence f(x) = qe it l3 (ii) Mean of X, w= [x f@)ax 1 3 2 J ¥f@) aes fxs) drt [xf de 1 3 a me = 04 fx g@-Dide+0 = 3 fx—Dide 1 1 90 Probability and Statistics & Complex Variables [EMIT # ¥ is continuous random variable and ¥ = aX + 6, prove that E(Y) = a E(X) + b and V(¥) = a?.M(X), where V stands for variance and a, b are constants. (i) [INTUO, (A) Dec. 09 (Set No.2), (H), (A) Nov. 2010, Nov. 2011, Apr. 2012, (H) Sept. 2017] :() By definition, FQ) = E@ax+d) = flrs sayar [rE f 9 Sepa OO Jxsordces | food = aE +50) [since total probability is unity] = aE(X)+b From (i), we have B(Y)= aE (X)+6 ~(@) whereY = aX +b w= Q) (2) (1) gives Y- EY) = @ [X- E)] Squaring, [Y- E(Y)P = a? [X- EOP Taking Expectation of both sides, we get EXLY- EP} = @ EX - EP} WY) = @V(X) If Xis a continuous random variable and & is a constant, then prove that (i) Var (X + k) = Var (X) (ii) Var (kX) = Var (X) — [JNTU 2006, 2007(Set No. 1)] Solution : By definition, 2 Var (X) = f Ps) dr — | J vse oo oo 2 ( Var (X+ kK) = f +e? rej ooh fo 2 = J (x? + 2k +k?) ree-| J x f(x) detk J reve] 2 J Pf) de+2k J x f(x) det k? | J «sect] : i severe Random Variables 1 = E(X?) + 2k B(X) + P - (EQ) + HP = E(X°) + 2k E(X) + - [EXP - 2k EX) - P = EX) - (BOOP = Var (X) 2 J vepne-| Jere “| (ii) Var (kX) 2 =k jenna] roa R [zur {E00} =P Var (X) Forthe continuous random variable X whose probability density function cx (2-2), if 0 0, find () k (ii) Mean (iii) Variance [INTU 2005, 2005S, 2007, (K) Dec. 2013 (Set No. 4)] (OR) Is f(x) = prem when x20 can be regarded as a probability function for a continuous random variable? If, so find Mean and Variance of the random variable. [JNTU (H) Nov. 2015] (Hint : In order that f(x) should be a probability function [ f(x) dx=1] Sol n: (i) Wehave J f(x)dx=1 + [ePetae=1 Co x20) 0 ie, k [PCe*)- 220) 42-9], = 1, Using Bernoulli’s Rule. ie,k fre"Gta2rtay}? =a 1 he, k+2)=1 or k= > (i) Mean =E(x)= J f(x)de = [ered a =k [P@e)-3:7 ("+ 6x(-e°*) - 66 Using Bernoulli's Rule = kee 43 +6x+6)] =k [0 +6) = 6k reo (9 Random Variables 93 (ii) Variance = E(x) -[E@) = J esoyar? fe. bo? ede -B)= & [reas 0 = & [xt (2) 4x3 (e*) 4 12x? (-e*) 2 x(-« em) 424 |" 9 be Al The trouble shooting capability of an IC chip in a circuit is a random variable X whose distribution function is given by 1 bette +4e' +123? +24e+24]5 -9 [: 1 = [0 +24)-9=12-9=3 0, for x <3 FO =) 15 torx>3 x where x denote the number of years. Find the probability that the IC chip will work properly (i Less than 8 years (ii) Beyond 8 years (iii) Between Sto 7 years —_(iv) Anywhere from 2 to 5 years [JNTU (K) Nov. 2009 (Set No.4)] Solution : We have F(x)=P(X 8) = 1-P(X<8) = 1-0.8594 = 0.1406 (ii) P(Ssxs7)= ra F)- (1-3 - (1-3) z 9(Z-4) 24%. _ 0.1763 © "(as 49) ~ 25x49 P(20 LO) = Vo forrso Determine (i) k (ii) the distribution function for X. i) Wehave f f(x)de =1 0 © I 2 ies f fejde+f fayde=1 ie, 0+ f2kxe* del 3 ca 0 ® 5 e he,k fxe*de=l ie, k fe‘de=1 (Putting 2 = 1) o 0 or k(-€ rat or-k-1)=1 k=l (ii) The distribution function is FIX) = P(X sx) = f sadr=0 ifx <0 n 4 and F(X)= J far+[ f@ dt, ifx> 0 a 2 ae tet = 04 fretdr = fe“ du (Putting 2 =u) a - (ef =F -1 K 1-e™ if x>0 ©* | o otherwise Suppose a continuous random variable X has the probability density f(0)=K (1-2) for 0 0 fle) = 9 Ox<0 If the total production is 12 million KW-hours, determine the probability that there is power cut (shortage) on any given day. [JNTU Jan. 2008, (K) May 2010S] Solution : Probability that the power consumed is between 0 to 12 is 2 PQ 12) = 1-PO0 a 3 . 1 7 3] =q@v=p If the probability density function of X is given by x/2,for01 JO=\ 0, otherwise ** SO-] 0, itxst Weave F(x) = | £6) de Itx>1, Fa)= J Sar i Random Variables 99 Ifx<1, F@)=0 0, if xsi SFO) 1M ittcrce x ‘The probability density function is = JkQGx? =), -1sx<2 0, else where Find the value of k and find P(-1<.x<0) [JNTU Apr. 2009 (Set No. 4)] © 4 2 2 Solution : Since f f(x)dx=1,wehave | f(x)de+ f f(x)de+ | f(dx=1 * a a 2 ie, fo aes | K@e -yaes fo de =1 » 4 2 Thus y=46 137, -1sxs2 0, else where 0 eo, He P(-lsx<0)= \dx =— = = lence P(-1 Xis F(x) ae 0, x<0 Random Variables 101 Find (i) the density function f(x), (ii) mean and (iif) variance of the density function. [NTU (K) May 2010 (Set No.2)] Solution : (i) The density function (x) is given by f(x) = en soy=}2° "7° 0,x<0 (i) Mean = E(x) = frr@ars Jared fo seas teu -04fxterar-1fremar 0 2 24 [lee] foment 1 1 =-gl0-O+d]=5 (iii) Variance = [ °f(x)ax—1? = Mf Hseyaes [seyae? a ye =[ee Mae “if fehl) ]-4 “2 =2 4 8) |, 64 Yen(4 2,1 YL 1 =- eeixee]| 2 2 2° 2" 4) ), 64 Lp ed =n gle* (ar +2r40] -F =—lho- ae == gl0-Or orn 2 FRUITS TEY 1£ X is a continuous random variable with p.d.f f(x) =1* 0S*S4 P P 0,clsewhere If p(asxsl)= S. find the value of ‘a’, [JNTU (K) May 2010 (Set No.3)] 102 Probability and Statistics & Complex Variables Solution: scien plosxsi)=fi- | fs a= =he Par= 8 (2) 19 1-a_19 19 ={£]-2= 3), 81 3 1946 toa aed se)= lie -D,-1sxs3 0, elsewhere 1 5 Find the value of '" and oS Sxs 3) [JNTU (K) May 2010 (Set No. 4)] Solution : In order that /(x) should be a probability density function ff@yae=1 ie, | feddr+ | fo) de+f f@)de=1 = 5 3 siz 5 5 x) =3[(62t-3) (a2 1} hi2 28} 2. 3 2. -3([8 | + 28124 24. 3 x6. 19 28" mm 56 Random Variables 103 [RRIIIENEY !* is the continuous random variable whose density function is x ifO0 Kan! 9), = K(tan"!oo—tan""(-20)=1 o«(8-(3)] =kre-l. By definition, the distribution function is given by [: x) =) ftan“! x= tan’ ‘con =4( tan x8), x ® 2 F(x)= J fdr = f lel+x [RENEE The length of time (in minutes) that a certain lady speaks on the telephone is found to be random phenomenon, with a probability function specified by the function Ae*, for x20 0, otherwise Sx) (i) Find the value of A that makes f(x) a probability density function. (ii) What is the probability that the number of minutes that she will take over the phone is more than 10 minutes? [JNTUCK) Dec. 2013 (Set No. 2)] Solution: (i) In order that f(x) should be a probability density function, [rest ie, freyac=1 or [ae deat 4 0 0 => -5A(€*-e%)=1 > -SA(0-1)=1 = 5A=1 oF (ii) PEK >10)= f fa) de= fAe*S dx =A 0 io Random Variables 105 1. REVIEW QUESTIONS Define expectation for discrete and continuous random variables. (EXERCISE 2(B)) The probability density function f (x) of a continuous random variable is given by kx, for00 | OSe@=) SC)=) 4 elsewhere 8 8 density function. 0, elsewhere a 106 Probability and Statistics & Complex Variables 5. “The diameter of an electric cable is assumed to be continuous random variable with probability density function f(x) = 6x (1 — x), 0 0 Le, ¥30,6>0 @S@) = )4 WS@=)% 0, elsewhere 0, elsewhere Find the mean and variance of X. 8. _Xisa continuous random variable with probability density function given by ke, for 0S x<2 S (x) =) 2k for 2sx<4 k(6—x), for 4sx<6 Find & and mean of the density function. [JNTU 2003, 2004] 9. Find the standard deviation of the probability density function 00,p>0 0, otherwise re{ Find & and mean value of X. [INTU (K) Nov. 2011 (Set No. 3)] Random Variables 107 13. Let f (x) = 3x7, when 0 a) (ii) PX > b) = 0.05. [JNTU 2004 (Set No. 2), (KX) Nov. 2011 (Set No. 1)] Aost0,-lexel 14, IEf(x)= 4 2 represents the density of a random variable X, find 0, elsewhere the mean and standard deviation of X. [INTU 20045 (Set No. 4)] 15. The probability density function of a random variable X is given by kee (x1), forex <4 LC) = | o, otherwise . 28 Given that P (I< X<3)= 5°. Find the value of k. 1 Gif OSx<8 16. Let X bea continuous random variable with distribution : /0)=48° °S* 0,elsewhere Find (i) P(2< X <5) (ii) PG0, the distribution is positively skewed (ii) If ps <0, the distribution is negatively skewed (iii) If uy =0, the distribution is symmetrical 4, The Fourth Moment about mean measures Kurtosis of a frequency distribution. 5. Skewness and Kurtosis of a distribution are calculated from j1,}13 and py and are given by 110 241 Probability and Statistics & Complex Variables Skewness =p, = 4 oo () and Kurtosis =B2 (2) ate From (1), we have By =+ 43, cd So the sign of skewness depends upon the sign of 4. RAW MOMENTS (OR) MOMENTS ABOUT ARBITRARY ORIGIN (ANY POINT) ‘When the actual mean of a distribution is a fraction, then it is difficult to calculate moments by using the above formulae. In such a case, we first compute moments about an arbitrary origin ‘A’ and then convert these moments into the moments about the actual mean. Moments about the arbitrary origin are called RAW moments and are denoted by the symbol y,. Thus pj stands for first moment about arbitrary origin (or about any point) A and so on. Ml. The r moment about the point A is given by TK Ay Lf df where d;=X;-A In the case of class interval frequency distribution, uy =D /(@i)" where d » where C= length of interval. c Relation Between Moments about Mean in terms of Moments about any point and vice versa 1 = Hi nj =0 Be =H (uj)? hy =H} —3h gn] +200)" 4g + 6n3(H))? ~3(uj)* wi =X-A Bh) =H +(H))* H5 = Hs +3HH +)? y= hg + Ataay + 6 (uf)? + (H)* Random Variables 1 These formulae should be committed to memory. They enable us to find the moments about any point, once the mean and the moments about mean are known Purpose of Moments : From the above discussion, we conclude that the moments are very useful tools and techniques in statistical analysis. The measure of the characteristics of a frequency distribution such as mean, median, mode, standard deviation, skewness and Kurtosis are directly connected with the first four central moments of a distribution. They can be obtained as given below. @ Mean =A+ny (ii) Variance =p, cy 3 1 (iii) Skewness =p, = Coefficient of Skewness : SK=+-43, where the sign of skewness SK is w determined from the sign of yy. (iv) Kurtosis =B2 = 2.12 MOMENTS FOR CONTINUOUS PROBABILITY DISTRIBUTION 2 (yy, (about origin) = fx f(x) dx ’ (ii), (about the point x= A) = f(x- AY’ fx) de In particular from (i), we have ’ yj (about origin) = Mean = f x f(x)de 2 ny = fered 8 (iii) p, (about mean) = {(x—mean)’ f(x) dx 112 Probability and Statistics & Complex Variables OLVED wee [BRIDES Fine the first four moments for the set of numbers 2, 4, 6, 8 Solution: Here N= number of items =4 2444648 _ 20 a 4 We have mean, Let d, =x)-¥=1,-5 Now we have the following table : Table : Computation of Moments Numbers -5 a? dp a 2 9 27 81 4 -1 1 -1 1 6 1 1 1 1 8 3 9 27 81 Lx, =20 Zd,=0 | Sd? =20 Lait = 164 Moments about mean are Hence the first four moments are 0,5,0,41 Calculate the first four moments of the following distribution about the ‘mean x:]0]1]2[3]4][5] 6 [7/8 £:[1[8 [28 [56 [70 [56 | 28 Also evaluate B, and By. Solution : We shall first calculate the moments about the arbitrary p\ Let x-Aax-4 Random Variables 113 ‘Table : Computation of Moments about A=4 x f d=x-4| fd fa Sa? fat 0 1 =4 =4 16 = 64 236 1 8 3 4 n -216 648 2 28 -2 -56 12 224 448 3 56 a 56 56 -56 56 4 10 0 0 0 0 0 3 56 1 56 56 56 56 6 28 2 56 2 204 448 7 8 3 4 n 216 648 8 1 4 4 16 “4 256 Total | EN= f=256 | Za =0 | Sd =0| 5 fa? =512| 5 fa? =0 | Dsfa* = 2816 Moments about the point A=4 are Moments about the actual mean are H=0 eee hy =n) (4)? =2-0=2 Hy = 5 —3ugnj + 2(H{)? =-3(2)(0) +200) = 0 Hg = Hh Aun +65 (Hi)? — 34a) =11-4(0)(0) + 6(2\(0)-30) =11 Nature of Distribution => The distribution is symmetrical. 114 Probability and Statistics & Complex Variables 78 (<3) = The distribution is Platykurtic. Hence the given distribution is symmetrical and Platykurtic. Note : 1. If Bz >3, the curve is more peaked than the normal curve (i.e., neither flat nor peaked) is called leptokurtic. 2. If By =3, the given distribution is normal. Calculate the first four moments of the following distribution about an arbitrary origin : Class interval : | 60-62 | 63-65 | 66-68 | 69-71 | 72-74 Frequency : 5 18 42 27 8 Also find the moments about the mean. Solution : Let us take the assumed mean A = 67 Let d A= 5-67 ‘Table : Computation of First Four Moments about Arbitrary O1 Classes Mid Freq. | d; | Sid; | fd? | fd? | fat Value (x) fi 60-62 61 5 -6 -30 180 1080 | 6480 63-65 64 18 3 54 162 ~ 486 | 1458 66-68 61 2 | 0 o | o 0 0 69-71 70 27 3 81 243 729 2187 72-14 7B 8 6 48 288 1728 | 10368 Total NES Eds | La? | E Aid? | Dae =100 =45 | =873 | =891 |=20493 The first four Moments about the arbitrary origin (i.., arbitrary mean) are . » Df? _873_ Hi Ha = N= 799 873 Lid) _ 891 _ , fed? _ 20493 _ BENT = 799 “891 HN Too 20493 ‘The first four moments about the actual mean are Random Variables 115 m1 =0 Hy = nh — (uf)? =8.73-(0.45)? =8.5275 or 8.53 Bs = Hh —3pdu{ + 2(u{)? = 8.91-3(8.73)(0.45) + 2(0.45)> =8.91-11.7855 + 0.1822 = 2.693 Ha =H 4h} +63 (4)? ~304)* = 204.93—4(8.91)(0.45) + 6(8.73)(0.45)° -3(0.45)* 104.93 -16.038 + 10.607 -0.123 = 199.376 [EEDIIIERD Compute the first four moments about the mean for the following distribution ‘Marks | 0-10 | 10-20 | 20-30 | 30-40 | 40-50 | 50-60 | 60-70 No.of Students| 8 | 12 20 | 30 15 10 5 Also evaluate B, and By Solution : We shall first find the moments about an arbitrary origin, then converts them into moments about actual mean, Let us take the assumed mean, A=35,C=10. m-A_m-35 10 ‘Table : Computation of Moments about Arbitrary Mean Then d Marks No. of Mid fa fa | fae | sat ‘Students (/) |value (m) 0-10 8 5 3 24 72 216 648 10-20 12 1S 2 24 48 96 192 20-30 20 25 1 -20 20 -20 20 30-40 30 35 0 0 0 0 0 40-50 15 45 1 15 15 15 15 50-60 10 55 2 20 40 80 160 60-70 5 65 3 15 45 135 405 Total | N=5S Zed | ry? | vp | opt =100 = 240 |=—102 | =1440 116 Probabilly and Statistics & Complex Variables ,_ Sd? 240 me # C2 = FRx 100= 240 rf 102 10000 = 144000 100 Moments about the actual mean are wy =0 Hy =H) — (i)? = 240—(-1.8)? = 240-3.24 = 236.76 Hs = 14 —3n ju +2(uj)> =-1020 + 1296 11.664 = 264.336 by = HY —4njn§ + 6(n)) Hd —30H{)* 144000 — 7344 + 4665 —31.49 = 141289.51 #3 _ 264.3367 13 (236.76) Thus By 86 40 are 1, 16 and —40. Show that the mean =3, the variance =15 and p15 Itis given that arbitrary origin A =2,n{ =1,u5 =16,u3 = ‘Thus mean =3 -ay Now yp BOCA 2 BOTA) 1g ig Random Variables 1. 2 2 pas () = 24-(X)? =24-9=15 i.e., Variance =15 Now Hs =H} —3n jh) +2(q{)° = =-40-48+2=-86 40-3 (1) (16) +2.()3 (REVIEW QUESTIONS) EW QUESTIONS 1. Define moments. List the relation between the moments about mean (central moments) in terms of moments about any arbitrary point and vice-versa. 2. Distinguish between raw and central moments. Write down the formulae for obtaining first four central moments in terms of raw moments. 3._What are two types of moments ? CEXERCISE 2(C)) IRCISE 2(C. Compute the first four moments about the mean from the following data : [2314/5 0 TF Bislan x:{/1/2]3 |4 [5 16 |7/8|9 @®) [Fife fe [25 [30 [22 [9 [5 [2 x: [5 [10] 15 | 20 [25 [30] 35 (ii) |Fz78 fis | 20 [32 [23 [17 | 5 Calculate the first four moments of the following distribution about the mean Class Interval | 0-10 | 10-20 | 20-30 | 30-40 | 40-50 Frequency [2 2 3 2 1 The first four moments of a distribution about the value 4 of the variable are 1, 4, 10 and 45. Show that the mean =5, Variance =3 and jt; =0. (i. The first four central moments of a distribution are 0, 2.5, 0.7 and 18.75. Calculate By and Bp. (di) The values of ,41),115 and jg are 0, 9.2, 3.6 and 1.22 respectively. Find out the skewness and Kurtosis of the distribution. The first four moments about the working mean 28.5 of a distribution are 0.294, 7.144, 14.409 and 454.98. Calculate the moments about the mean. Also evaluate "7 B, and B2. (ie., Skewness and Kurtosis) 118 Probability and Statistics & Complex Variables CANSWERS ) 1. (i) 0,1.262,0.722,3.795 (ii) 2. 0,156,144,49392 4. (i) 0.0313. i) 0.129,1.4 5. 28,794, 7.058, 36.151, 408.738; B, =3.717, By =8.205 2.13 MOMENT GENERATING FUNCTION —_[JNTU (K) June 2015, (1) May 2017] It is a tool used to calculate the higher moments. The moment generating function of a random variable X, about the origin, whose probability density funetion f(x) is given by Mo(®) or Mx (1) =E (e") Ze" p(x) incase of discrete random variable J e*Ac(2) im case of continuous random variable Since M,(t) is used to generate moments, it is known as moment generating function. Now we shall discuss how M,(t) is used to generate the moments. 2 Wehave of =14 24 © *. My()=E(e")= =H) +5 eee) ECs. a . SLM HS tot Se the coefficient of — in the rn expansion of My(1). Thus Mix) generates moments and that is why it is called the moment where 1; is the r moment of X about the origin i.e. E(x") =) generating function. In order to find y', we have to differentiate the moment generating function My(#), ‘'r’ times with respect to ‘f and substitute 1=0 [jee Random Variables 119 In general, (The moment generating function of a discrete random variable X about the point x= a is ) defined as the expected value of e"*~®) and is denoted by M, (1) .M,(0) = My(0) (about x=a) =E fe] =e) feat tt eat HE) Leu rma) (ea) tat Seay + aan fy 7 where 1 =E{(x—a)"} is ther” moment about the point x=a. Moment Generating Function of Continuous Variate (ii) If fx(x) is the density function of a continuous random variable X, then the moment generating function of this continuous probability distribution about x =a is given by M,(#) or Mx(¢) (about x=a) = f BO) F(x) de Note : Mj(t)=e“Mo(t) or eo“ My (0) ie., The m.g.f about the point 'a'=e"*" (m.g.f about the o1 OBJECTIVE TYPE QUESTIONS 1. Maximum value of probability is in) 2. The function f(x) = kx in 0 0 7. HF) =) 9 elsewhere #8 the probability density function, then k= __ 120 10. u. 12. 13. 14. 16. 17. 18. 19. 20. 2. 22. 23. Probability and Statistics & Complex Variables 2 If) =x+ Z.x= 1, 2, 3,4, Sis the probability distribution of a discrete random variable, then k Iff (x) = A x in 0 0 is a probability density function, then k = If f(x) = 2 €°*, x > 0 is a probability density function, then P(X > 0.5) = If X is uniformly distributed in (a,b), then E(X)= The relation between probability density function and cumulative density function of a random variable is 1 1 mw 2 34 4 = Fy 1 5 6 6 5 1A 8-5 200 1 3 WS 11.2 2 5 ae wu 2 1s. 4 w + 10 a) as 25 1 18. 1 19. 1 2, 2 " . "2 EE) 22, 3.083 23. a E(X,) + b EX) 0,1 25. V(X) 26. 0 21. @ 8 aw, 4 30. 2 31. 3 5 2 4 122 32. 35. Probability and Statistics & Complex Variables 1 1 1 4 33.0 = 34. (a+b) 3 2 3at) FQ)= j Slxde ~ PREVIOUS GATE QUESTIONS Let X be a real - valued random variable with E{X] and E[X?]denoting the mean values of X and X?, respectively. The relation which always holds true is (A) (E[X])? > E[X?] (B) E{X?]> (E[X])? IGATE2014(EC)] (© ELX?]=(E(XI? (D) ELX?] > EEX)? Let X be a random variable which is uniformly chosen from the set of positive odd numbers less than 100. The expectation, E[X] is IGATE2014 (EQ) Consider a dice with the property that the probability of a face with n dots showing up is proportional to n. The probability of the face with three dots showing up is, IG: 2014 (EE)} Let X be a random variable with probability density function (0.2, for|x|<1 f(x) =4 0.1, forl<|x|<4 0, otherwise The probability P(0.5 0 and 0 otherwise. The expected value of the function gx(x)=e**"* is [GATE2015 (IN)] Let the probability density function of a random variable X be given as : 3 43x, 0 p P . Sale = 50 S*u(x)+ae"u(—x) where u(x) is the unit step function. ‘Then the value of ‘a’ and Prob [X <0], respectively are, [GATE 2016(EE)] 1 1 1 1 A) 2 @) 4 © 2 ©) 4g 124 14, 15. 16. 17. 18. 19. Probability and Statistics & Complex Variables Probability density function of a random variable X is given below. _ fo.2sif lex<5 FO)=1o, otherwise P(X <4) is [GATE2016(CE)] a3 pt © + p) + wa ®; OG O; If f(x) and g(x) are two probability density functions, x S4L-asx<0 a S@)=)_* = ~Gthosxsa and 8(%) Es1osxsa 0, otherwise O,otherwise which one of the following statements is true ? IGATE2016(CE)] (A) Mean of f(x) and g(x) are same ; variance of f(x) and g(x) are same (B) Mean of f(x) and g(x) are same; variance of f(x) and g(x) are different. (C) Mean of f(x) and g(x) are different; variance of f(x) and g(x) are same. (D) Mean of f(x) and g(x) are different; variance of f(x) and g(x) are different. A probability density function on the interval [a,1] is given by + and outside this interval the value of the function is zero. The value of of a is IGATE 2016(CS)| A six-face fair dice is rolled a large number of times. The mean value of the outcomes is [GATE 2017(ME)] Forthe function f(x)=a +bx,0< x <1 , tobea valid probability density function, which one of the following statements is correct. [GATE2017(CB) (A) a=1,b=4 — (B) a=0.5,b=1 (C) a=0,b=1 (D) a=1,b=-1 Jf) The graph of a function (x) is shown in the figure For f(x) to bea valid probability density function, the value of his [GATE2018(CE)] 1 2 A) BZ on (D)3 Random Variables 125 20. 21, 22, 23, 24, 25, 26. X and Y are two independent random variables with variance | and 2, respectively. Let z=X-Y. The variance of z is [GATE 2018(IN)} (ayo @)1 ©2 (D)3 The probability distribution for a discrete random variable X is given below xX] 1 27 3] 4 P(X) [03 [0.4 [02 [0.1 The expectation value of X is [GATE 2018 (BD) If X and Y are random variables such that E(2X+Y)=0 and E(X+2Y)=33, then E(X)+E(Y)= _ IGATE 2019 (EQ)] ‘The mean square of a zero-mean random process is “where K is Boltzmann's temperature, and c is a capacitance. The standard deviation of the random process is KT VRT KT ¢ A (B) ; Oye Og [GATE 2019(EE)] Ifx is the mean of data 3,x,2 and 4, then the mode is, [GATE 2019 (ME-Set 2)] Let T be a full binary tree with 8 leaves (A full binary tree has every level full). Suppose ‘two leaves a and b of T are chosen uniformly and independently at random. The expected value of the distance between a and b in T (i. the number of edges in the unique path between a and b) is [GATE 2019 (CS)] A ‘The function p(x) is given by P(*)=—> where A and } are constants with j1>1 and x 1) ("here =(q+ | a =mp Ce q+ p=) Thus the mean of the Binomial Distribution is mp . Probability Distributions 135 vo m(n=1) (0-2) eng Ee ae 2 (n-2 =n(n=1p* [s (3) 2a om (Refer nj) a =¥ xe- = =n(n—1)p?(q+ py"? +mp=n(n—lp? +np_— (2 9+ Pp =1) ny = E03) =D pap = o3() xonx “SoS = VeG-) G-243G- deal?) 5 = de- Ix a(t)or qn #435 a(e- of are Soa( "nat & fa) entn=nin-a0° (3) 0 3g" 4 3n(n—Np “xr 3p 2g" + np a3 =m(n=n=2)p°(q + py" +3n(n=1)p*(g+ py"? + =n(n—-1(n—2)p? +3n(n-1)p? tmp Co g+ p=) Similarly, ny =) = > «()) pea" * mo 5 5 [x(a —1)(-2)(x-3) + 6x(x = 1(x-2) + 7x(0— Deal]prer = =n(n—I(n—2(n 3) p* + 6n(n—-1)(n—2)p* + Tn(n—1)p? + np (on simplification) Central Moments of Binomial Distribution m=0 ba = -(n})? =[(n—Dp? + np ]-(np? = (0? =n)p? ¢mp—n? p? =n? p? —np? + np—n? p? = -np? +np = np (1~ p)=npq 136 Probability and Statistics & Complex Variables Hs = #5 3b +2004) =[n(n-1)(n=2)p? +3n(n=1)p? + np]-3 [n(n =I) p? + np] inp] + Amp) = np [-3np? +3np + 2p -3p +1-3npq] = np Bnp(1- p)+2p? -3p +1-3npq] = np npg +2p? -3p +1-3npq] =np (2p? ~3p +1) = np (2p? -2p +(1-p)] =np (2p? -2p+4]=np[-2p(l- p)+4 =np(-2pq+4]= npg (1-2p) =npq (q+ p-2p)=np4q (q~ P) Hes = 14 4 + 6nd (HH)? —30H))* = npq [1+3 (n—2)pq] (on simplification) 3.7. BINOMIAL FREQUENCY DISTRIBUTION Ifn independent trials constitute one experiment and this experiment is repeated N times, then the frequency of r successes is N. "C, p’ q"~". Since the probabilities of 0, 1, 2, ssey Fy vy MI Successes in 7 trials are given by the terms of the binomial expansion of (q +p)", therefore in NV sets of n trials the theoretical frequencies of 0, 1, 2, coy Myon successes will be given by the terms of expansion of N(q + p)".The possible number of successes and their frequencies is called a Binomial Frequency Distribution, [REMIND 4 fair coin is tossed six times. Find the probability of getting four heads Solution : p = Probability of getting a head = 1/2 q = Probability of not getting head = 1/2 and n =6,r=4 We know that P(r)= "C, p'g"~" 1) (rt _ 6 6 1 _ 1s «perc (5) (3) = arm($) = S8-e = gnome Determine the probability of getting the sum 6 exactly 3 times in 7 throws with a pair of fair dice. Sol n : In a single throw of a pair of fair dice, a sum of 6 can occur in 5 ways : (1, 5), (5, 1), (2, 4), (4, 2) and (3, 3) out of 6 * 6 = 36 ways. 3 Thus p = Probability of occurrence of 6 in one throw = == 36 53 3636 Probability Distributions 137 n= Number of trials = 7 <. Probability of getting 6 exactly thrice in 7 throws 3 a1y8 4 5 (31)* _ 35(125) 31 =1G4pq? ="C, (3) (2) - 35029). G0" ap Y = 0.0516 (nearly). A die is thrown 6 times. If getting an even number is a success, find the probabilities of (i) at least one success (ii) < 3 successes (iif) 4 successes. Solution : In a single throw of a die, an even number can occur in 3 ways out of 6 ways. Thus p = Probability of occurrence of an even number in one throw = n= Number of trials = 6 @ P(rzi=1-P(r=0) 6 1 1 6 -1-(3) = 1 Gz 0nd (i) P(r $3)= P(r = 0) + P(r= 1) + P(r = 2) + Pr =3) I) Se 21 2 J) Clot SGC, + Cs) = 35 = 0.6565 oe (ALY _ 15 «.(3) (3) wero Ten coins are thrown simultaneously. Find the probability of getting atleast (i) seven heads [JNTU 1999, 2007, 20085, (K) May 2013 (Set No. 2),(H) Ill yr. Nov. 2015] (ii) six heads [JNU (A) Dec, 2009, (K) May 2013 (Set No. 2)] (iii) one head ——_{JNTU(H) II yr. Nov. 2015] Solution: p = Probability of getting a head = 1/2 q = Probability of not getting a head = 1/2 The probability of getting r heads in a throw of 10 coins is (ii) P(r=4) + 7 \loor PX =r)= p(= "e(3) (3) 57 =0,1,2,.., 10 () Probability of getting at least seven heads is given by P(X> 1) = P(X = 7) + P(X = 8) + P(X = 9) + P(X = 10) DT DOO O Qo) (ii) This is left as an exercise to the student. 138 Probability and Statistics & Complex Variables P(r=0) 0410 =e (1) (1) 23-41 eo(5) (3) 2 ‘Two dice are thrown five times. Find the probability of getting 7 as sum (iii) P (atleast 1 head) = p(r21) (i) at least once (ii) exactly two times iii) P(1) =1- P(X =0) “J 8) (ii) P (exactly two times) = P(X=2) = *e,(2) 4 s =10 =-2(2) & 6 \6 (ii) PU 4) = PIX = 4) + POX = 5) = 0.25924 °C,(0.6 [From (i)] = 0.2592 + 0.07776 = 0.337 Probability Distributions 139 If 3 of 20 tyres are defective and 4 of them are randomly chosen for inspection, what is the probability that only one of the defective tyre will be included ? [JNTU 20008] 3 Solution: Let p= Probability of a defective tyre (success) = 35 Then q = Probability of a non-defective tyre 37 te 20° 20 Given n = 4 The probability that exactly one tyre will be defective gel 3 = Pea n= 6H) (B) = Gyr 03685 The incidence of an occupational disease in an industry is such that the workers have a 20% chance of suffering from it. What is the probability that out of 6 workers chosen at random, four or more will suffer from the disease ? [JNTU 2001 S, (K) Nov. 2011 (Set No. 4)] Solution : The probability of a worker suffering from disease, p = 20% = 0.2 «. The probability that no worker suffering from disease, q = 1 - p = 0.8 No. of workers, n = 6 The probability that four or more workers will suffer from disease =P(X2 4) = P(X = 4) + PX = 5) + PIX = 6) C4(0.2)* (0.8)? + °C,(0.2)° (0.8) + °C, (0.2) (0.2)*[15(0.8)? + 6(0.2) (0.8) + (0.2)"] = (0.2)* [9.6 + 0.96 + 0.4] = 0.0175 Determine the binomial distribution for which the mean is 4 and variance 3, [JNTU 1999, (H) Sept. 2017] Solution : Given mean of the distribution = 4 i.e., np =4 and variance of the distribution = 3. ie., npg = 3 (2) + (I) gives q = 3/4 3 I-g=1-7 “p From (1), 1 = z. The given Binomial Distribution has parameters n= 16 and p = 1/4 Hence the Binomial distribution is Isc ger 7 =0,1,2,.050 PXX=a=pey=y AG) 4) rr 0, otherwise 140 Probability and Statistics & Complex Variables -25, which is not an Note : To find mode of the distribution, we have (n + 1) p = 2 - integer. Hence the unique mode of the distribution = integral part of (n + 1)p ie., 4.25 =4 If the probability of a defective bolt is 1/8 , find (The mean (ii) The variance for the distribution of defective bolts of 640. [INTU (41) Nov. 2009 (Set No. 4)] Solution: We are given p= The probability of a defective bolt = 1/8 and n= 640. 640 :, Mean of the distribution, = np = =~ = 80 Also Hence variance of the distribution = npq = (np)q = wg = 80x. gr PRRIMIINTEY 12 256 sets of 12 tosses of a coin, in how many cases one can expect 8 heads and 4 tails. [JNTU 2003, 2004 (Set No. 4)] Sal ition : The probability of Deeps ahead, p= 1/2 . The probability of getting a tail, q= 1/2. Here n= 12 The probability of getting 8 heads and 4 tails in 12 trails a LR, 1 1 = px=8)= "o(4) (4 par=s="G(4) (2) _ 4 1)? _ 495 - 2 (2)? _ The expected number of such cases in 256 sets 4 49: = 256 x P(X=8)= 2 ae ee 30.9375 ~ 31 If the probability of success is 1/20, how many trials are necessary in order that the probability of at least one success is just greater than 1/2. 1 _19 Solution : Here p = 809-1 oan0} Let n be the required number of trails so that the probability of at least one success wen) sume (J > 1 ve, (BY (since equal probability for boys and girls) Number of children, n = 5 The probability distribution is PO) = "C,prar ye ase (LY (4 S (3) (3) 1 = yp SC, per family jo 5 P (3 boys) = P(r= 3) = p(B) + $C, = 39 = 76 Per family Thus for 800 families the probal ry of number of families having 3 boys = Po) = 250 families 1 L P ( girls) = P(no boys) = P(r = 0) = p(0) = 55 » Cy = 35 per family Thus for 800 families, the probability of number of families having 5 girls 1 ‘ 433 (800) = 25 families P (either 2 or 3 boys)= P(r = 2) + P(r = 3) = p(2) + p(3) 1, 1 Bats 25 3p 7g er family acs <. Expected number of families with 2 or 3 boys = 2 (800) = 500 families P (at least one boy) = P(r=1)+ P(r =2)+ P(r=3)+ P(r =4)+ P(r=5) 1 31 po S404 10454 = 35 1 a3 CG $50, +90, 450, + C5 . Expected number of families with atleast one boy = 600) =775 This is left as an exercise to the reader.

You might also like